PHARMACOLOGY, TOXICOLOGY & PHARMACOKINETICS (PINK)

Ace your homework & exams now with Quizwiz!

The insecticide Azinphos-methyl can cause death when ingested at a dose of 20 mg/kg body wt. What is its toxicity rating? A.1 B. 2 C. 5 D. 5 E. 6

D. 5

Which of the following agent is most useful in the management of acetaminophen poisoning? a. N-acetylcysteine b. S-carboxymethylcysteine c. Dantrolene d. Atropine e. Sodium Bicarbonate

a. N-acetylcysteine

Which of the following calcium channel blockers is most useful in arrythmia therapy? a. Verapamil b. Diltiazem c. Nifedipine d. Lidocaine e. Propafenone

a. Verapamil

Epoprostenol, a drug that is being used in the management of primary pulmonary hypertension is an analogue of which eicosanoid? a. PGD2 b. PGI2 c. PGE1 d. PGE2 e. PGF2-alpha

b. PGI2

Which of the following agents for the heart failure may be associated with arrythmia, thrombocytopenia, aand hepatotoxicity which limit their use in patients with chronic CHF? A. Bipyridines B. Digitalis glycosides C. Beta-1 agonists D. ACE inhibitors E. Angiotensin II antagonists

A. Bipyridines

.Toxidromes of poisoning with chlorinated hydrocarbons primarily consists of which of the following manifestations? A. CNS stimulation B. CNS depression C. Bone marrow suppression D. Cardiac rhythm disturbance E. Hepatic failure

A. CNS stimulation

The manifestations of oxalate poisoning are best managed with which of the following antidotes? A. Calcium gluconate B. Fomepizole C. Glucagon D. Sodium bicarbonate E. Atropine

A. Calcium gluconate

In synaptic neurotransmission, which of the following ions plays a significant role in neurotransmitter release a. Sodium b. Potassium c. Magnesium d. Calcium e. Chloride

d. Calcium

.Which of the following agents exhibit a primary concentration dependent bactericidal activity wherein the rate and extent of killing increases with increasing concentrations of drug? A. Amikacin B. Vancomycin C. Clindamycin D. Ceftriaxone E. Piperacillin

A. Amikacin

The minimum dose that produces the maximum achievable response is known as a. Potency b. ED50 c. LD50 d. Ceiling dose e. Minimum effective dose

d. Ceiling dose

Royal jelly, a popular nutritional supplement form honeybee has been reported to cause which of the following adverse effects? A. Hepatotoxicity B. CNS depression C. CNS stimulation D. Anaphylaxis E. Hemorrhagic gastritis

D. Anaphylaxis

Which of the following findings in a patient taking Phenytoin would warrant reduction in the dose? A. Gingival Hyperplasia B. Nystagmus C. Osteomalacia D. Ataxia E. Hirsutism

D. Ataxia

What is the primary use of the drug Flumazenil? A. Antagonizes the action of the Benzodiazepines B. Stimulates BZ-1 receptor similar to benzodiazepines. C. Antagonizes the action of Barbiturates D. Inhibits the 5HT-1A receptor E. Stimulates the GABA-A receptor

A. Antagonizes the action of the Benzodiazepines

Autoinduction of the metabolism is a phenomenon that leads to decrease in the usual half-life of a given drug that the dose adjustments must be made within the first month of therapy to ensure therapeutic drug levels are reached, and reduction of dosage must be made once metabolism has stabilized. Such is seen with which of the following drugs? A. Carbamazepine B. Phenytoin C. Phenobarbital D. Valproic acid E. Primidone

A. Carbamazepine

The use of activated charcoal as a means of eliminating a drug taken in overdose is applicable for which of the following drugs? A. Carbamazepine B. Ethanol C. Ferrous sulfate D. Lithium carbonate E. Potassium chloride

A. Carbamazepine

Which of the following aliphatic hydrocarbons is considered as the most hepatotoxic? A. Carbon monoxide B. Chloroform C. Trichloroethylene D. Tetrachloroethylene E. Methyl chloroform

A. Carbon monoxide

Which of the following agents is a first generation cephalosporin that is given intravenously? A. Cefazolin B. Cephalexin C. Cephadrine D. Cefadroxil E. Cephapirin

A. Cefazolin

Which of the following agents is considered as fourth generation cephalosporin? A. Cefepime B. Cefotaxime C. Cefoperazone D. Cefixime E. Moxolactam

A. Cefepime

Which of the following cephalosporins is associated with hypoprothrombinema and has increased risk of bleeding disorders? A. Cefoperazone B. Cefoxitin C. Cefuroxime D. Ceftriaxone E. Cefotaxime

A. Cefoperazone

Which of the following agents is considered as first line drug in the management of neisseria gonorrhea infection? A. Ceftriaxone B .Cefuroxime C.Penicillin G D. Amoxicillin E. Trimethoprim-Sulfamethoxazole

A. Ceftriaxone

Which of the following local anesthetics is a substrate for metabolic degradation by the enzyme butyrylcholinesterase? A. Chlorprocaine B. Etidocaine C. Ropivacaine D. Lidocaine E. Bupivacaine

A. Chlorprocaine

Which of the following antidiabetic drugs has the longest half-life? A. Chlorpropramide B. Tolazamide C. Tolbutamide D. Glyburide E. Glimepiride

A. Chlorpropramide

Which of the following drugs can be given orally to treat infections caused by Peudomonas aeruginosa? A. Ciprofloxacin B. Ceftazidime C. Piperacillin D. Clindamycin E. Azithromycin

A. Ciprofloxacin

Which of the following agents is considered as the standard drug for the initial management of Pneumocystis jiroveci (carinii) pneumonia in patients with AIDS? A. Cotrimoxazole B. Pentamidine C. Doxycycline D. Meronidazole E. Emetine

A. Cotrimoxazole

What is the most common side effect seen with the use of carbamazepine which requires adjustment in the dose of the drug? A. Diplopia B. Drowsiness C. Hyponatremia D. Vomiting E. Leukopenia

A. Diplopia

Which of the following agents can cause a state of drunkenness followed a day later by severe anioin gap metabolic acidosis? A. Ethylene glycol B. Ethanol C. Gamma-hydroxybutyrate D. Lorazepam E. Diazepam

A. Ethylene glycol

Therapy with HMGCoA reductase inhibitors is associated with Rhabdomyolysis and myositis. These complications which can lead to acute tubular necrosis and renal failure have been reported to be augmented with co-administration of which drugs. A. Fenofibrates and other fibrates B. Cholestyramine and other bile acid binding resins C. Captopril and other ACE inhibitors D. Tranylcypromine and other MAOI E. Methampethamine and other Norepinephrine releasers

A. Fenofibrates and other fibrates

Which of the currently available inhalation anesthetic is the most potent based in the minimum Alveolar concentration? A. Halothane B. Desflurane C. Nitrous Oxide D. Sevoflurane E. Enflurane

A. Halothane

The ability of a chemical agent to cause injury in a given situation or setting is called? A. Hazard B. Risk C. Exposure D. Toxicity E. Threshold

A. Hazard

Which of the following manifestations is most consistent with acute overdose of Iron, particularly in children? A. Hemorrhagic gastroenteritis B. Acute hepatic necrosis C. Acute pulmonary edema D. Encephalopathy and convulsions E. Acute Gingivostomatitis

A. Hemorrhagic gastroenteritis

Which of the following drugs or drug combinations is appropriate maintenance therapy for patient with primary adrenal insufficiency? A. Hydrocortisone + Fludrocortisone B. Dexamethasone + Hydrocortisone C. Hydrocortisone D. Prednisone E. Betamethasone

A. Hydrocortisone + Fludrocortisone

A 50 year old male with Generalized Tonic-clonic seizure was initially given Carbamazepine as maintenance therapy. He however developed adverse response to drug in form of Stevens-Johnsons syndrome. Which of the following agents may be safed to used as an alternative treatment for the seizure? I. Valproic acid II. Phenytoin III. Phenobarbital A. I only B. II only C. I and III D. II and III E. I, II and III

A. I only

A patient presenting with encephalopathy, abdominal colic, hemolytic anemia and an elevated liver enzymes several days after exposure to large amounts of a metallic poison may be suffering from what poisoning? I. Acute inorganic lead poisoning II. Acute Inorganic arsenic poisoning III. Acute mercury A. I only B. II only C. I and III D. II and III E. I, II and III

A. I only

Which of the following antidiabetic drugs produced control of blood sugar by promoting directly or indirectly the release of insulin by the remaining functional beta cells of the pancreas? I. Meglitinides II. Biguanides III. Thiazolidinediones derivatives A. I only B. II only C. I and III D. II and III E. I, II and III

A. I only

Which of the following describe/s the appropriate management of acute nicotine toxicity? I. Anticonvulsant therapy mainly involves the use of benzodiazepine like Diazepam II. Epinephrine is given to increase the blood pressure III. Neostigmine is given to counteract the neuromuscular blockade A. I only B. II only C. I and III D. II and III E. I, II and III

A. I only

.Which of the following is the preferred empiric treatment for patients suspected or proved to have sysytemic fungal infection of any etiology? A. Itraconazole B. Amphotericin B C. Fluconazole D. Flucytosine E. Ketoconazole

B. Amphotericin B

Which of the following interventions may be appropriate to monitor or treat thionamide-associated agranulocytosis? I. Discontinuing the offending drug usually reverses the side-effect II. Monitoring the WBC is necessary for the first three months of therapy III. Switching of therapy from one thionamide to another may reverse the effect A. I only B. II only C. I and III D. II and III E. I, II and III

A. I only

Which of the following may be an expected complication with the use of Aminoglycosides? I. Neuromuscular blockade II. Thrombotic Thrombocytopenic purpura III. Inhibition of the CYP3A4 enzyme A. I only B. II only C. I and III D. II and III E. I, II and III

A. I only

Which of the following statements correctly describes the characteristics of Aminoglycosides? I. Exhibit synergistic activity with beta-lactam antibiotics II. May be given both orally and parenterally for systemic activity III. Readily penetrate the blood brain barrier reaching adequate concentrations in the CSF A. I only B. II only C. I and III D. II and III E. I, II and III

A. I only

Which of the following statements correctly describes the thyroid hormones? I. T3 is better absorbed after oral administration than T4 II. Peripheral deiodination of T3 converts it to more active T4 III. Hypothyroidism increases the clearance of both hormones leading to an decrease in their half-lives A. I only B. II only C. I and III D. II and III E. I, II and III

A. I only

What is the Primary role if Zinc in commercial insulin preparations? A. Improve stability and shelf-life of insulin B. Improve the solubility of insulin C. Enhance the activity of insulin in receptors D. Facilitate absorption of insulin from subcutaneous tissue E. Enhance the activity of suspending agents

A. Improve stability and shelf-life of insulin

What is the mechanism of action of Atorvastatin? A. Inhibits the first committed step in the biosynthesis of cholesterol B. Serves as ligand for the nuclear transcription called peroxisome proliferators-activated receptor alpha C. Stimulates the activity of lipoprotein lipase leading to lipolysis of lipoprotein triglyceride D. Inhibits the reuptake of bile acids leading to loss of negative feedback on the alpha-hydroxylase enzyme E. Prevents oxidation of LDL cholesterol

A. Inhibits the first committed step in the biosynthesis of cholesterol

Which of the following new insulin preparations has a characteristics release pattern that shows no peak and a plateau serum insulin level that is maintained for about 24 hours? A. Insulin Glargine B. Insulin Lispro C. insulin Aspart D. Ultralente Insulin E. NPH Insulin

A. Insulin Glargine

Based on recent studies what is currently considered as the most probable explanation for the action inhalation anesthetic? A. Interaction with the molecular components of the GABA-A receptor B. Interaction and interference with the activity of the fast inward sodium channels C. Non-specific interaction with the lipid matrix of the nerve membrane D. Inhibition of the interaction with Glutamate of the NMDA receptor E. Inactivation of the Ligand-gated Potassium, channels

A. Interaction with the molecular components of the GABA-A receptor

Which of the following drugs given for tuberculosis is metabolized by the liver enzyme N-acetyltransferase whose expression is genetically determined may vary in different populations? A. Isoniazid B. Rifampicin C. Ethambutol D. Pyrazinamide E. Streptomycin

A. Isoniazid

Which of the following drugs is most useful in controlling the bradykinesia of Parkinsonism? A. Levodopa B. Bromocriptine C. Amantadine D. Selegiline E. Entacapone

A. Levodopa

Which of the following agents is currently the preferred drug for the initial management of Status Epilepticus? A. Lorazepam B. Diazepam C. Phenobarbital D. Phenytoin E. Valproic acid

A. Lorazepam

Poisonings with antipsychotics are rarely fatal. Which of the following agents is associated with a higher incidence of fatal poisoning? A. Mesoridazine B. Chlorpromazine C. Trifluphenazine D. Haloperidol E. Molindone

A. Mesoridazine

Which of the following rugs may be useful to manage complication of exposures to nitrogen oxides and nitrobenzene? A. Methylene blue B. Ethanol C. Calcium gluconate D. Activated charcoal E. Penicillamine

A. Methylene blue

When large doses of a local anesthetic must be administered, the risk of developing seizures can be significantly reduced with the used of which of the following agents as pre-medication? A. Midazolam B. Thiopental C. Succinyl choline D. Propofol E. Diphenhydramine

A. Midazolam

Which of the following agents is associated with significant anterogade amnestic effect which is advantageous in a number of medical and surgical procedures that may discomfort to the patient? A. Midazolam B. Zolpidem C. Buspirone D. Phenobarbital E. Thiopental

A. Midazolam

Which of the following Aminoglycosides as the most ototoxic in terms of tis side-effect on the cochlear apparatus? A. Neomycin B. Amikacin C. Streptomycin D. Tobramycin E. Netilmicin

A. Neomycin

Which of the following Aminoglycosides is considered as the most nephrotoxic? A. Neomycin B. Amikacin C. Streptomycin D. Tobramycin E. Netilmicin

A. Neomycin

Which of the following drugs is associated with cutaneous vasodilation and warmth sensation after an initial dose which may be reduced with the use of 300mg dose Aspirin taken an hour before the dose? A. Niacin B. Fenofibrate C. Atorvastatin D. Cholestyramine E. Probucol

A. Niacin

Which of the following toxicants is known to accumulate slowly in the lungs by an active process and causes lung edema, alveolitis, and progressive pulmonary fibrosis days to weeks after an exposure? A. Paraquat B. Scopolamine C. Parathion D. Dioxin E. Nicotine

A. Paraquat

Which of the following agents is considered as the drug of choice for most cestodal and trematodal infection? A. Praziquantel B. Mebendazole C. Niclosamide D. Albendazole E. Metronidazole

A. Praziquantel

Which of the following agents can be safely used for a pregnant women with urinary tract infection? A. Cotrimixazole B. Ampicillin C. Norfloxacin D. Doxycycline E. Chloramphenicol

B. Ampicillin

Which of the following correctly provides the sequence of symptoms associated with carbon monoxide poisoning? A. Psychomotor impairment > headache > confusion > tachycardia coma B. Convulsions > headache > tachycardia > loss of visual acuity > coma C. Tachycardia > confusion > psychomotor impairment > loss of visual acuity > coma D. Coma > confusion > headache > tachycardia > loss of visual acuity E. Convulsions > loss of visual acuity > confusion > tachycardia coma

A. Psychomotor > impairment > headache > confusion > tachycardia > coma

Which of the following agents is given with the Sulfonamides for its synergistic effect with the latter or inhibiting folic acid synthesis in protozoal cells than mammalian or bacterial cells? A. Pyrimethamine B. Trimethoprim C. Silver D. Mafenide E. Salicylic acid

A. Pyrimethamine

MAO-A is the amine oxidase primarily responsible for the metabolism of Norepinephrine, Serotonin, and tyramine, while MAO-B is the specific for the metabolism of Dopamine. Which of the following agents is a selective reversible MAO-A inhibitor whose advantage is a lesser risk of causing hypertensive crisis with tyramine foods? A. Selegiline B. Nefazodone C. Moclobemide D. Trazodone E. Imipramine

A. Selegiline

Ocreotide is a drug used for the treatment of such conditions as acromegaly, gastrinoma, and glucagonoma. What hormone is ocreotide an analog of? A. Somatostatin B. Growth hormone C. Prolactin D. Oxytocin E. Dopamine

A. Somatostatin

The so-called Isoxazolyl penicillins are primarily indicated for the treatment of infection caused by what organism? A. Staphylococcus aureus B. Bacteroides fragilis C. Escherichia coli D. Haemophilus influenzae E. Streptococcus peumoniae

A. Staphylococcus aureus

Which of the following agents may predispose the neonate premature babies to kernicterus when given to this subset of patients? A. Sulfamethoxazole B. Ethambutol C. Chloramphenicol D. Doxycycline E. Ciprofloxacin

A. Sulfamethoxazole

Which of the following agents is most likely to cause deposits in the retina which can lead to blindness necessitating a baseline and regular ophthalmologic examination when administered? A. Thioridazine B. Chlorpromazine C. Loxapine D. Trifluphenzaine E. Molindone

A. Thioridazine

Which of the following drugs for psychosis is most commonly associated with prolongation of QT interval on ECG which can lead to fatal arrhythmias? A. Thioridazine B. Mesoridazine C. Haloperidol D. Clozapine E. Loxapine

A. Thioridazine

During the maintenance therapy with Levodopa, patients are advsed to take the drug on an empty stomach. Why is this advice given? A. To minimize the risk of aspiration in as much as the drug can cause nausea and vomiting B. Oral bioavailability of the drug is reduced as Levodopa can be found to food particles in the GIT C. Bile acids released in response to food can interfere with the absorption of the drug D. Large neutral amino acids in food interfere with the drugs' transport acroos the blood brain barrier E. Food can induce expression of DOPA decarboxylase which can limit the drugs CNS bioavailability

A. To minimize the risk of aspiration in as much as the drug can cause nausea and vomiting

In terms of potency, which of the drugs listed below is expected to be the most potent? A. Trifluorophenazine B. Thioridazine C. Piperacetazine D. trifluopromazine E. Chlorpromazine

A. Trifluorophenazine

Although the clinical value of resistance reversal is not established, there are certain drugs that have been shown to reverse chloroquine-resistance in malaria. Which of the following is an exemple of resistance reversing drugs? A. Verapamil B. Propranolol C. Calcium gluconate D. Primaquine E. Diphenhydramine

A. Verapamil

Which of the following drugs may be given to type 1 DM patients as a combination therapy with Insulin? A. Voglibose B. Repaglinide C. Nateglinide D. Chlorpropamide E. Glyburide

A. Voglibose

In monitoring response to therapy with methimazole, which of the following findings taken 3 months after initiating therapy best indicates good response to therapy? A. a normal TSH B. a normal T4 C. a lower than normal T4 D. a lower than normal TSH E. a higher than normal TSH

A. a normal TSH

Livedo Reticularis, a vascular cutaneous reaction characterized by a reddish-purple, fish net- patterm mottling of the Upper/Lower extremities, is a side effect associated with which of the following agents? A. amantadine B. Di[henhydramine C. Levodopa D. Pramipexole E. Selegiline

A. amantadine

Yusho disease documented in Japan in the 1960's is associated with exposure to which of the following toxicants? A.Polychlorinated biphenyls B. Paraquat C. Nicotine D. Aconite E. Mercury

A.Polychlorinated biphenyls

Which of the following Beta-Lactam antibiotic is called a monobactam? A. Moxolactam B. Aztreonam C. Meropenem D. Loracarbef E. Ticarcillin

B. Aztreonam

What is the primary repository site of lead in the body of an adult containing about 90% of the total body lead burden? A. Liver B. Bone C. Brain D. Gastrointestinal tract E. Skin and its appendages

B. Bone

A patient is receiving 50mg of Propylthiouracil three times a day. If the plan is to shift her medication to methimazole, what dose of methimazole should you recommended? A. 5 mg once daily B. 15 mg once daily C. 30 mg once daily D. 45 mg once daily E. 60 mg once daily

B. 15 mg once daily

What is the acceptable dose for isoniazid in the treatment of TB? A. 1 mg/kg/day B. 5mg/k/day C. 10mg/k/day D. 15mg/k/day E. 20mg/k/day

B. 5mg/k/day

What is considered as the most common cause of industrial lead poisoning? A. Absorption of Lead via the gastric mucosa B. Absorption of lead via the respiratory tract C. Absorption of lead through the skin D. Absorption of lead from the antiknock gasoline E. Absorption of lead from the intestines

B. Absorption of lead via the respiratory tract

Which of the following is the most common problem encountered in female patients using Progestin only oral contraceptive pills? A. Mastalgia B. Breakthrough bleeding C.Increase in pigmentation D. Hirsutism E. hypertension

B. Breakthrough bleeding

What is the most important toxidrome of acute exposure to benzene? A. CNS Stimulation B. CNS depression C. Bone marrow injury D. Peripheral neuropathy E. Leukemia

B. CNS depression

Which of the following drugs reversibly binds to the 30S subunit of the bacterial ribosome and inhibits the peptidyl transferase step of the protein synthesis? A. Aminoglycosides B. Chloramphenicol C. Linezolid D. Tetracyclines E. Macrolides

B. Chloramphenicol

Which of the following drugs is most commonly associated with seizures? A. Thioridazine B. Chlorpromazine C. Loxapine D. Trifluphenazine E. Molindone

B. Chlorpromazine

What is the rationale behind the use of long-acting benzodiazepine like chlordiazepoxide and diazepam in the management of alcohol withdrawal? A. The benzodiazepines produce a paradoxical reaction among chronic alcoholics B. Cross-dependence develops among sedative hypnotics C. Increase in renal clearance of alcohol D. Increase in non-CYP mediated metabolism of alcohol

B. Cross-dependence develops among sedative hypnotics

The Anti-psychotic drug Clozapine belongs to what chemical class of agents? A. Dibenzoxazepine B. Dibenzodiazepine C. Benzisoxazole D. Fluorobenzyindole E. Thienobenzodiazepine

B. Dibenzodiazepine

filariasis an infection which can lead to Lymphatic obstructive disease is caused by Wuchreria bancrofti and Brugia malayi. Eradication of microfilaria can be accomplished by the use of which of the following agents? A. Praziquantel B. Diethycarabamazine citrate C. Niclosamide D. Albendazole E. Metronidazole

B. Diethycarabamazine citrate

What is the mechanism of action of Bromocriptine in the management of Parkinsonism? A. Direct antagonist of D2 receptors B. Direct agonist of D2 receptors C. Direct antagonist of 5HT3 receptors D. Direct agonist of 5HT3 receptors E. Metabolized in the CNS to Dopamine

B. Direct agonist of D2 receptors

Which of the following is most appropriate treatment for the management of cardiac manifestations of theophylline or Caffeine overdose? A. Verapamil B. Esmolol C. Digoxin D. Magnesium sulfate E. Adenosine

B. Esmolol

Which of the following agents primarily cause lowering of the VLDL and is thus useful for hypertiglyceridemia? A. Niacinamide B. Fenofibrate C. Simvastatin D. Probucol E. Cholestyramine

B. Fenofibrate

Which of the following is the common complication associated with the alpha-glucosidase inhibitors? A. Cardiac arrhythmia B. Flatulence C. Congestive heart failure D. Hepatotoxicity E. Interstitial Nephritis

B. Flatulence

What is the physiologic dose of glucocorticoid? A. Hydrocortisone dose less than 10-20 mg/day B. Hydrocortisone dose greater than 10-20mg/day C. Prednisone dose ranging from 10-20mg/day D. Prednisone dose less than 10-20mg/day E. Prednisone dose less than 10-20mg/day

B. Hydrocortisone dose greater than 10-20mg/day

Alteration of pH can alter renal elimination of certain toxins. Which of the following toxicants may be safely eliminated by either acidification or alkalinization of urine? I. Phencyclidine II. Salicylates III. Amphetamines A. I only B. II only C. I and III D. II and III E. I, II and III

B. II only

Serotonin syndrome which resembles malignant hyperthermia in symptomatology occurs when SSRI's like Fluoxetine and Sertraline are given concurrently with I. TCA's like Nortriptyline II. MAOI's like Phenelzine III. Heterocyclics like Maprotiline A. I only B. II only C. I and III D. II and III E. I, II and III

B. II only

What is the primary use of the drug Desmopressin? I. Management of nephrogenic Diabetes insipidus II. Management of Central Diabetes Insipidus III. Control of gastrointestinal bleeding A. I only B. II only C. I and III D. II and III E. I, II and III

B. II only

Which of the following agents interventions is/are appropriate in the management of poisoning with the Phenothiazine antipsychotic? I. Gastric lavage is unnecessary one hour after ingestion of tablets II. Activated charcoal is given followed by saline cathartics III. Epinephrine is the preferred pressor agent for persistent hypertension A. I only B. II only C. I and III D. II and III E. I, II and III

B. II only

Which of the following antibacterial agents primarily work by inhibiting cell wall synthesis? I.Polymyxin B II. Vancomycin III. Streptpgramins A. I only B. II only C. I and III D. II and III E. I, II and III

B. II only

Which of the following cautions must be observed when iodides are given to treat hypothyroidism? I. Iodides must be initiated before thionamide therapy II.Iodides must be avoided if treatment with radioactive iodine is likely III. Iodides are the preffered drugs for chronic therapy during pregnancy A. I only B. II only C. I and III D. II and III E. I, II and III

B. II only

Which of the following correctly describes the nomal physiologic hormone release? I. GnRH is released continuously by the hypothalamus to induce secretion of FSH and LH by the pituitary II. ACTH release occurs in pulses that peak iin the early morning hours and after meals III. The release of cortisol is constant throughout the day maintaining a level that varies little with meals A. I only B. II only C. I and III D. II and III E. I, II and III

B. II only

Which of the following mechanisms may lead to inhibition of platelet aggregation? I. Activation of antithrombin III II. Thromboxane synthesis inhibition III. Guanylyl cyclase inhibition A. I only B. II only C. I and II D. II and III E. I, II and III

B. II only

Which of the following statements is/are correct regarding the toxicity profile and the management of overdose of most drugs used for treatments of seizures? I. The most dangerous effect is cardiotoxicity II. Alkalinization of the urine to hasten elimination is usually effective III. Stimulants are useful antidotes A. I only B. II only C. I and III D. II and III E. I. II and III

B. II only

Which of the following statements is/are correct regarding the toxicology of the carbamate insecticides? I. Inhibition of the cholinesterase enzyme is reversed easily with the use of pralidoxime II. The clinical approach to management is similar to organophosphate poisoning III. They are considered as persistent toxicants with great impact in the environment A. I only B. II only C. I and III D. II and III E. I, II and III

B. II only

which of the following intravenous agents can only be used to induced anesthesias but not to maintain anesthesia? I. Propofol II. Thiopental III. Ketamine A. I only B. II only C. I and III D. II and III E. I, II and III

B. II only

What is the mechanism of action of quinolone antibacterial agents? A. Inhibit the incorporation of GABA into Dihydropteroate B. Inhibit the bacterial enzyme Topoisomerase II C. Inhibit the DNA dependent RNA polymerase of bacteria D. Inhibit the Transpeptidation process in bacterial cellwall synthesis E. Inhibit bacterial dihydrofolate reductase

B. Inhibit the bacterial enzyme Topoisomerase II

What is the common mechanism of action of the Local anesthetics when given in their usual doses? A. Inhibition of Ligand gated sodium channel B. Inhibition of the voltage gated sodium channel C. Stimulate outward potassium flux D. Stimulate calcium extrusion from the cell E. Inhibit the sodium, potassium ATPase Pump

B. Inhibition of the voltage gated sodium channel

Which of the following agents is considered to be the most neurotoxic when given as a spinal anesthetics causing a transient reticular irritation? A. Procaine B. Lidocaine C. Bupivacaine D. Mepivacaine E.Prilocaine

B. Lidocaine

A psychiatric patient on medications develops tremor, thyroid enlargement, and leukocytosis. The drug he is taking is most likely A. Chlorpromazine B. Lithium C. Desipramine D. Fluoxetine E. Haloperidol

B. Lithium

Patients on Statins are generally advised to take the medication at night. What is the basis for this? A. Maximal absorption occurs at night B. Maximal cholesterol de-novo synthesis occurs at night C. Minimize postural hypotension expected with Statins D. Minimize Syncopal attacks with the first dose of statins E. LDL release occurs at night

B. Maximal cholesterol de-novo synthesis occurs at night

Erethism, a behavioral pattern characterized by change in mood from shyness, withdrawal and depression with explosive anger or blushing, is seen as a manifestation of metal poisoning. It can be seen with other findings such as tremors progressing to choreiform movements of limbs and gingivostomatitis. Which of the following metals may be responsible for these manifestations? A. Lead B. Mercury C. Iron D. Copper E. Arsenic

B. Mercury

Which of the following agents has acivity against anaerobic organisms against most protozoan infection? A. Clindamycin B. Metronidazole C. Chloramphenicol D. Doxycycline

B. Metronidazole

Which of the following agents is most useful for the management of acetaminophen toxicity? A. S-carboxymethylcysteine B. N-acetylcysteine C.Atropine D. Ethanol E. Oxygen

B. N-acetylcysteine

Which of the following metabolites of local anesthetics when formed in the body in significant amounts can cause oxidation of the iron in hemoglobin in the ferric form? A. Glycinexylidide B. Ortho-toluidine C. para amino benzoic acid D. Benzoic acid E. Demethylamine

B. Ortho-toluidine

Which of the following drugs has been historically associated with the development of Creutzfeldt-Jakob disease, a fatal neurodegenerative disease caused by prions? A. Pancreas derived insulin B. Pituitary derived growth hormone C. Thyroid hormone extract D. Recombinant growth hormone E. Cortisol

B. Pituitary derived growth hormone

.Which of the following side-effects of Lithium can be seen even in usual doses thus by itself is NOT considered a sign of Lithium toxicity ? A. Coarse hand tremors B. Polyuria C. hyperreflexia D. Confusion E. ataxia

B. Polyuria

What is the treatment of choice for the management of severe falciparum malaria? A. Chloroquine phosphate B. Quinidine gluconate C. Artemether D. Artesunate E. Doxycycline

B. Quinidine gluconate

The second generation cephalosporin includes the Cephamycins. What is the distinctive advantage of the cephamycin over the rest of the second generation cephalosporin? A. they have greater selectivity against Staphylococcus aureus B. They have average for anaerobic bacteria C. They are effective against MRSAs D. They have greater spectrum of activity including Peudomonas aeruginosa E. They can be given both orally and parenterally

B. They have average for anaerobic bacteria

Which of the following agents is associated with development of thrombotic thrombocytopenic purpura? A. Aspirin B. Ticlodipine C. Tirobifan D. Eptifibatide E. Cilostazol

B. Ticlodipine

Which of the following Benzodiazepines has the shortest half-life and a rapid onset of action? A. Lorazepam B. Triazolam C. Oxazepam D. Diazepam E. Prazepam

B. Triazolam

Which of the following agent is associated with the idiosyncratic hepatotoxicity when given for the treatment of seizure in children less than 2 years of age and in patients with multiple medications? A. Gabapentin B. Valproic acid C. Phenytoin D. Primidone E. Topimarate

B. Valproic acid

Historically, a "universal antidote" was employed in the management of poisoning. Recent studies have shown that the two of the components of this so-called universal antidote have no significant efficacy. What are the components of this so-called "universal antidote"? A. activated charcoal, calcium oxide, tannic acid B. activated charcoal, magnesium oxide, tannic acid C. activated charcoal, milk of magnesia, milk D. Milk of magnesia, tannic acid, strong tea E. activated charcoal, milk, ipecac

B. activated charcoal, magnesium oxide, tannic acid

A 20-year old female is receiving Hydrocortisone 20 mg IV three times a day. If the plan is to discharge the patient with an equivalent dose of Oral Prednisone, what dose of Prednisone will you recommend? A. 5 mg once a day B. 10mg once a day C. 10 mg in the morning and 5 g in the afternoon D. 15 mg in the morning and 10 mg in the afternoon E. 20 mg in the morning and 15 mg in the afternoon

C. 10 mg in the morning and 5 g in the afternoon

What is the dose for rifampicin in the treatment of TB? A. 1 mg/kg/day B.5mg/k/day C. 10mg/k/day D. 15mg/k/day E.20mg/k/day

C. 10mg/k/day

What is the minimum fatal dose of nicotine, equivalent to 2 sticks of cigarettes? A. 10mg B. 20 mg C. 40mg D. 80 mg E. 160 mg

C. 40mg

Paraquat is described as having a toxicity rating of 4. What are the probable human lethal dosages with this rating? A. 5,000-50,000mg/kg body weight B. 500- 5,000mg/kg body wt C. 50- 500mg/kg body wt D. 5-50mg/kg body wt E. not more than 5 mg/kg body wt

C. 50- 500mg/kg body wt

What is Entacapone? A. A selective MAO-A inhibitor B. A selective MAO-B inhibitor C. A COMT inhibitor D. A Dopamine-3 receptor Agonist E. A Dopamine-2 receptor agonist

C. A COMT inhibitor

Which of the following agents is classified as an inhibitor of glycoprotein IIb/IIIa and is useful in patients with acute coronary syndrome and those undergoing percutaneous coronary intervention? A. Clopidogrel B. Ticlodipine C. Abciximab D. Cilostazol E. Urokinase

C. Abciximab

Which of the following drugs classified as diuretics maybe useful in epileptic women who have seizures exacerbations at the time of menses? A. Hydrochlorothiazide B. Metolazone C. Acetazolamide D. Chlorthalidone E. Torsemide

C. Acetazolamide

Imipramine has been found to be useful in the treatment of nocturnal enuresis ( bed wetting). This clinical use explained by which of the following effects of the drug? A. Inhibition of reuptake of norepinephrine B. Inhibition reuptake of serotonin C. Anticholinergic effects D. Sedating effect E. Inhibition of reuptake of Dopamine

C. Anticholinergic effects

Statins are generally best taken at night. Which of the following statins is night dosing unnecessary? A. Lovastatin B. Simvastatin C. Atorvastatin D. Pravastatin E. Fluvastatin

C. Atorvastatin

Which of the following agents belong the cephalosporins group called cephamycins? A. Cefamandole B. Cefprozil C. Cefoxitin D. Loracarbef E. Cefaclor

C. Cefoxitin

Anticholinergic effect manifesting as loss of accommodation, difficulty in urination, constipation, and a toxic confusional state is most commonly observed with which of the following drugs? A. Haloperidol B. Sertindole C. Chlopromazine D. Loxapine E. Chlorpropthixene

C. Chlopromazine

Which of the following agents can penetrate inflamed meninges in sufficient concentration to be usedful for meningitis? A. Amikacin B. Tobramycin C. Chloramphenicol D. Aztreonam E. Gentamicin

C. Chloramphenicol

Which of the following benzodiazepines is a prodrug that is hydrolyzed to the active metabolite Nordiazepam in the stomach? A. Alprazolam B. Chlordiazepoxide C. Chlorazepate D. Diazepam E. Lorazepam

C. Chlorazepate

What is the recommended clinical use of Repaglinide in the management of Diabetes mellitus? A. Primarily given to control fasting blood glucose levels B. Improves tissue response to insulin C. Control post-prandial hyperglycemia D. Control overnight blood sugar E. Prevent absorption of carbohydrates

C. Control post-prandial hyperglycemia

What is the most important adverse effect of Dipyridamole when given to patients with ischemic heart disease? A. Significant thrombocytopenia B. Vasospasm of coronary vessels C. Coronary steal phenomenon D. Severe diarrhea E. intermittent claudication

C. Coronary steal phenomenon

Malignant Hyperthermia which may developed in susceptible individuals exposed to i9nhalational anesthetic is appropriately treated with which of the following agents? A. Diazepam B. Baclofen C. Dantrolene D, Thiamylal E. Haloperidol

C. Dantrolene

Which of the following agents is classified as a tricyclic antidepressant or 1st generation anti-depressants? A. Bupropion B Trazodone C. Desipramine D. Amoxapine E. Mirtazepine

C. Desipramine

Which of the following may be considered as mechanism/s of effect of Beta lactam antibiotics? I. Inhibition of activity of the transpeptidase enzyme II. Inhibition of formation of the 50s ribosomal subunit III. Binding to the so-called PBPs and related proteins in the cell membrane A. I only B. II only C. I and III D. II and III E. I, II and III

C. I and III

Which of the following statements correctly describes the pharmacokinetics of Phenytoin? A. Absorption of Phenytoin is constant regardless of the formulation B. Phenytoin is well absorbed after intramuscular injection C. Elimination of Phenytoin follows a first-order kinetics at very low blood levels D. A first order kinetics of metabolism is observed with Phenytoin at low and high blood levels E. Phenytoin is not significantly Protein bound so drug displacement interaction is not a major concern

C. Elimination of Phenytoin follows a first-order kinetics at very low blood levels

Retroperitoneal fibrosis is an observed complication seen with drugs that belong to what class of agents? A. Antiviral agents B. Tropane Alkaloids C. Ergot alkaloids D. Ethanolamine Anti-histamines E. Dopamine Agonist

C. Ergot alkaloids

What form of erythromycin has the best oral absorption? A. Erythromycin base B. Erythromycin ethylsuccinate C. Erythromycin estolate D. Erythromycin lactobionate E. Erythromycin stearate

C. Erythromycin estolate

A patient receiving sulfadiazine 4 grams per day may benefit with co-administration of which of the following agents as preventive therapy of sulfadiazine-related complications? I. Sodium bicarbonate II. Ammonium chloride III Folinic acid A. I only B. II only C. I and III D. II and III E. I, II and III

C. I and III

Ginkgo biloba is a common additive in a number of nutritional supplements. Which subset of patients should be cautioned against the use ginkgo-containing products? I. Patients receiving anticoagulation therapy II. Patients with Alzheimers Dementia III. Patients with history of seizures A. I only B. II only C. I and III D. II and III E. I, II and III

C. I and III

In which individuals should Biguanides be avoided as part of therapy for DM because of increased risk of lactic acidosis? I. Chronic alcoholics II. Coronary Artery disease III. Chronic hepatits A. I only B. II only C. I and III D. II and III E. I, II and III

C. I and III

The risk of stroke is especially increase when oral contraceptives are given to which subpopulation of patients? I. Age above 35 years old II. Use of progestin only pills III. Chronic smokers A. I only B. II only C. I and III D. II and III E. I, II and III

C. I and III

Which of the following agents can cause a high anion gap metabolic acidosis? I. Methanol II. Ethanol III. Metformin A. I only B. II only C. I and III D. II and III E. I, II and III

C. I and III

Which of the following agents may be useful for the treatment of ethylene glycol poisoning? I .Ethanol II. Glucagon III. Fomepizole A. I only B. II only C. I and III D. II and III E. I, II and III

C. I and III

Which of the following agents may be useful for the treatment of poisoning due to methanol? I .Ethanol II. Glucagon III. Fomepizole A. I only B. II only C. I and III D. II and III E. I, II and III

C. I and III

Which of the following anti-psychotics drug groups primarily inhibit the Dopamine-2 receptors in the brain? I. Phenothiazines II. Dibenzodiazepines III. Butyrophenones A. I only B. II only C. I and III D. II and III E. I, II and III

C. I and III

Which of the following anti-psychotics drug groups primarily inhibit the Dopamine-2 receptors in the brain? I. Phenothiazines II. Dibenzodiazepines III. Butyrophenones A. I only B. II only C. I and III D. II and III E. I, II and III

C. I and III

Which of the following cephalosporin is/are associated with Disulfiram-effect when alcohol is taken by patients who are on this/these antibiotics? I. Cefotetan II. Ceftazidime III. Cefamandole A. I only B. II only C. I and III D. II and III E. I, II and III

C. I and III

Which of the following correctly describes the clinical used of Carbidopa in the therapy of parkinsonism? I. Increased the CNS bioavailability of Levodopa II. Improve symptoms of bradykinesia III. Minimize nausea and vomiting associated with Levodopa A. I only B. II only C. I and III D. II and III E. I,II and III

C. I and III

Which of the following disease-causing organism is/are the aminoglycosides most useful either assingle agents or in combinations? I. Mycobacterium tuberculosis II.Mycobacterium avium intracellulare III. Staphylococcus aureus A. I only B. II only C. I and III D. II and III E. I, II and III

C. I and III

Which of the following drugs may enhance toxicity of Aminoglycosides if given with the latter concurrently to a patient? I.Penicillin G II. Furosemide III. Vancomycin A. I only B. II only C. I and III D. II and III E. I, II and III

C. I and III

Which of the following effects is/are consistent when cortisol in the body is less than 10 mg/day? I. Metabolism of carbohydrate, fat and proteins II. Inhibition of release of cytokines from macrophages III. Enhanced vascular and bronchial smooth muscle response to cathecolamiines A. I only B. II only C. I and III D. II and III E. I, II and III

C. I and III

Which of the following findings is/are consistent with acute nicotine toxicity? I. Hypertension II. Peripheral vasodilatation III. Cardiac arrhythmia A. I only B. II only C. I and III D. II and III E. I, II and III

C. I and III

Which of the following findings is/are consistent with acute nicotine toxicity? I. Hypertension II. Peripheral vasodilatation III. Cardiac arrhythmia A. I only B. II only C. I and III D. II and III E. I, II and III

C. I and III

Which of the following is/ are appropriate indications for using Glucocorticosteroids? I. In mothers who may have premature labor and delivery before 34 weeks of gestation II. In patients with secondary pulmonary tuberculosis prior to initiation of antitubercular therapy III. In patients who have recently undergone organ transplantation A. I only B. II only C. I and III D. II and III E. I, II and III

C. I and III

Which of the following macrolides can inhibit cytochrome P450 enzymes can cause an increase in the serum concentrations of drugs like theophylline and warfarin? I. Erythromycin II. Azithromycin III. Clarithromycin A. I only B. II only C. I and III D. II and III E. I, II and III

C. I and III

Which of the following oral anti-diabetic drugs is/are known to induce hypoglycemia among diabetic and euglycemic individuals? I. Sulfonylureas II. Biguanides III. Meglitinides A. I only B. II only C. I and III D. II and III E. I, II and III

C. I and III

Which of the following situations will contraindicate induction of vomiting as an intervention in cases of poisoning? I. Patient has ingested petroleum products II. Patient is less than 1 year old III. Patient is having convulsion A. I only B. II only C. I and III D. II and III E. I, II and III

C. I and III

Which of the following statement is correct regarding the use of a combination of the least potent Nitrous oxide with a second inhalation anesthetic? I. Reduction in the risk of the myocardial depression at a given depth of anesthesia compared to the more potent anesthetic given alone II. Facilitation of metabolism of the more potent anesthetic which may increase the risk of nephrotoxicity III. Reduction in the anesthetic requirement fot he more potane anesthetic due to a second gas effect. A. I only B. II only C. I and III D. II and III E. I. II and III

C. I and III

Which of the following statements correctly describes the properties of the drugs Minocycline and Doxycycline? I. They have the longest half-lives among the Tetracyclines II. Their absorption is not impaired by divalent cations like calcium and magnesium III. Their absorption is not impaired by food A. I only B. II only C. I and III D. II and III E. I, II and III

C. I and III

Which of the following statements is/are correct regarding Lente Insulin? I. It is a mixture of 30% semilente and 70% ultralente Insulin II. It is also known as NPH III. It is an intermediate-acting Insulin preparation A. I only B. II only C. I and III D. II and III E. I, II and III

C. I and III

Which of the following statements is/are correct regarding Penicillins? I Penicillins are not metabolized extensively by liver enzyme II. Probenecid increases Penicillin excretion by blocking renal tubule reabsorption III. Ampicillin undergoes enterohepatic cycling A. I only B. II only C. I and III D. II and III E. I, II and III

C. I and III

Which of the following statements is/are correct regarding intravenous Phenytoin? I. Cardiotoxicity is partly due to Propylene Glycol II. It is compatible with Dextrose containing liquid III. Fosphenytoin is more soluble and requires no solubilizing agent A. I only B. II only C. I and III D. II and III E. I, II and III

C. I and III

Which of the following statements is/are correct regarding the pharmacokinetics of lead? I. Lead can cross the placenta and pose a potential hazard to fetus II. Adults have greater degree of absorption of ingested lead than young children III. The major route of excretion of lead is through the urine A. I only B. II only C. I and III D. II and III E. I, II and III

C. I and III

Which of the following statements is/are correct regarding adrenal suppression as a complication of therapy with glucocorticosteroids? I. It is expected to occur when glucocorticosteroids therapy is extended beyond 2 weeks II. Dosage reduction when the therapy is to stopped must be done very slowly when the dose reaches replacement levels III. Patients must receive supplementary therapy at times of stress like surgery or trauma IV. Recovery of the hypothalamic-pituitary-adrenal axis is rapid after discontinuing chronic use of Dexamethasone V. Treatment with ACTH reduces the time required for the return of normal adrenal function A. I and III B. II and IV C. I, II and III D. I, III and V E. I, II ,III, IV and V

C. I, II and III

What is considered as the most common mode of exposure to poisons? A. inhalation B. Dermal exposure C. Ingestion D. Ophthalmic E. Bites

C. Ingestion

What is the mechanism of action of the drug Tranexamic acid? A. ADP inhibition epoxide B. Serine protease inhibition C. Inhibits plasminogen activation D. Inhibits reductase hydrolysis of Vitamin K E. Increase factor VIII activity

C. Inhibits plasminogen activation

Which of the following effects correctly describe the mechanism of action of arsenic as a poison? A. Inhibits generation of ATP by binding to cytochrome oxidase system in the electron transport chain B. Inhibits effective oxygen delivery by oxidizing the hemoglobin iron form ferrous to ferric C. Inhibits the enzyme activity by binding to the Sulfhydryl groups in the enzyme D. Alters DNA transcription by impairing synthesis of mRNA E. Intercalates with the DNA leading to failure of gene expression

C. Inhibits the enzyme activity by binding to the Sulfhydryl groups in the enzyme

Chronic excessive exposure to this metal can lead to deposition in various organs and tissues causing the development of conditions such secondary Diabetes mellitus, restrictive cardiomyopathy, and hepatic failure A. Lead B. Mercury C. Iron D. Copper E. Arsenic

C. Iron

Deferoxamine is primarily indicated for the treatment of poisoning with which of the following metals? A. Lead B. Mercury C. Iron D. Copper E. Arsenic

C. Iron

Which of the following mechanisms correctly describes the mechanism of action of aspirin as an inhibitor of platelet aggregation? A. Irreversible inhibition of platelet and endothelial cell cyclooxygenase B. Irreversible inhibition of endothelial cell cyclooxygenase C. Irreversible inhibition of platelet cyclooxygenase D. Reversible inhibition of platelet and endothelial cell cyclooxygenase E. Reversible inhibition of endothelial cell cyclooxygenase

C. Irreversible inhibition of platelet cyclooxygenase

Which of the following agents inhibits protein synthesis by binding at the 23S ribosomal RNA and preventing the formation of ribosome complex that initiates protein snthesis? A. Aminoglycosides B. Chloramphenicol C. Linezolid D. Tetracyclines E. Macrolides

C. Linezolid

Which of the following drugs when given to patients will necessitate weekly WBC monitoring for the first 6 months of therapy and every 3 weeks thereafter due to its propensity to cause agranulocytosis? A. thioridazine B.Chlopromazine C. Loxapine D. Clozapine E. Molindone

C. Loxapine D. Clozapine

The so-called "wearing-off" phenomenon encountered with the use of Levodopa-Carbidopa combination in Parkinsonism is partly due to the formation of which metabolite which can compete with the transport of Levodopa across the blood brain barrier? A. alpha-methyldopamine B. alpha-methylnorepinephrine C. alpha-methyldopa D. 3-O - methyldopa E. 3-0- methyldopamine

D. 3-O - methyldopa

A 50 year old male patient with type2 diabetes mellitus is maintain for the last 7 days of subcutaneous Isophane Insulin at 20 Units pre-breakfast and 10 units pre-supper. For the last three days, his capillary blood glucose levels have been consistently showing the following values: 110mg% ( pre-breakfast), 218mg% (2 hour post breakfast), 110 mg% (2 hour post lunch), and 110 mg% ( presupper). What recommendation will you make regarding his therapy? A. Increase his pre-breakfast insulin to 24 units while maintaining his pre-supper insulin at 10 units B. Increase his pre-breakfast insulin to 24 units and his pre-supper insulin to 12 units C. Maintain his pre-breakfast and pre-supper insulin doses but give insulin lispro 4 units pre-breakfast D. Maintain his pre-breakfast insulin at 20 units while increase his pre-supper insulin to 12 units E. No change in the insulin doses as all values are withih the acceptable range

C. Maintain his pre-breakfast and pre-supper insulin doses but give insulin lispro 4 units pre-breakfast

Among patients with renal impairment, the dose of antibiotics must be adjusted according to the creatinine clearance. Which of the following drugs does not require such adjustment in dosing for patients with decreased creatinine clearance? A. Imipenem B. Penicillin G C. Nafcillin D. Ceftazidime E. Cefuroxime

C. Nafcillin

Which of the following statements correctly describes the mechanism of action of the anxiolytic drug Buspirone? A. Selective binding to the BZ-1 receptor in the brain B. Antagonist of alpha receptors in the brain C. Partial agonist of 5HT-1A receptor in the CNS D. Inhibitor of MAO enzyme E. Antagonist of the BZ-1 receptor in the brain

C. Partial agonist of 5HT-1A receptor in the CNS

Which of the following antibiotics belong to the ureido penicillin class of agents? A. Carbenicillin B. Ticarcillin C. Peparicillin D. Bacampicillin E. Pivampicillin

C. Peparicillin

Which of the following drugs attacks plasmodiain exoerythrocytic stages and is an effective prophylactic agent against Plasmodium vivax and Plasmodium ovale? A. Quinine B. Chloroquine C. Primaquine D. Sulfadoxine E. Pyrimethamine

C. Primaquine

Which of the following drugs for seizures is metabolized to yield Phenobarbital? A. Lamotrigine B. Vigabatrin C. Primidone D. Topimarate E. Felbamate

C. Primidone

Allergic reaction is most likely to occur with which of the following agents? A. Isobucaine B. Prilocaine C. Procaine D. Lidocaine E. Mepivacaine

C. Procaine

Which of the following drug combinations is generally avoided? A. Simvastatin + Niacin B. Niacin + colestipol C. Simvastatin + Gemfibrozil D. Gemfibrozil + Colestipol E. Simvastatin + Colestipol

C. Simvastatin + Gemfibrozil

Which of the following Aminoglycosides is considered as the most vestibulotoxic? A. Neomycin B. Amikacin C. Streptomycin D. Tobramycin E. Netilmicin

C. Streptomycin

Which of the following antineoplastic agents is considered as cell cycle specific agent in terms of its action on cancer cells? A. Dactinomycin B. Daunorubicin C. Vincristine D. Cisplatin E. Mechlorethamine

C. Vincristine

What is the most common side effect associated with the used of tricyclic antidepressants? A. Sedation B. Tachycardia C. Weight gain D. Orthostatic hypotension E. Anticholinergic effect

C. Weight gain

The activity of Penicillin G is defined units. How many units does each milligram of crystalline sodium penicillin G contain? A. 1 B. 58 C. 180 D. 1600 E. 2600

D. 1600

Which of the following is considered as the most significant serious toxic effect of chronic exposure to benzene? A. Hepatotoxicity B. Nephrotoxicity C. Peripheral neuropathy D. Bone marrow injury E. Cardiotoxicity

D. Bone marrow injury

Exemestane is a steroidal hormone agent that binds to and irreversibly inactivate aromatase enzyme. What is the clinical use of the agent? A. Acute myelogenous leukemia B. Renal cell carcinoma C. Choriocarcinoma D. Breast cancer E. Colorectal cancer

D. Breast cancer

What is the preferred treatment for severe Verapamil overdose? A. Propranolol B. Sodium bicarbonate C. Epinephrine D. Calcium gluconate E. Naloxone

D. Calcium gluconate

Which of the following second generation cephalosporins can be given orally and parenterally? A. Cefaclor B. Cefuroxime axetil C. Cefotetan D. Cefprozil E. Cefoxitin

D. Cefprozil

Hepatolenticular degeneration is a result of chronic exposure to which metal? A. Lead B. Mercury C. Iron D. Copper E. Arsenic

D. Copper

Penicillamine, as a chelating agent, is primarily used for the treatment of poisoning with which of the following metals? A. Lead B. Mercury C. Iron D. Copper E. Arsenic

D. Copper

Which of the following tetracyclines is the drug of choice when tetracyclines are needed in a patient with renal insufficiency?? A. Oxytetracycline B. Chlortetracycline C. Methacycline D. Doxycyclie E. Minocycline

D. Doxycyclie

Which of the following antibiotics is contraindicated in children less than 9 years of age and in pregnant patients due to the risk of permanent staining of teeth and enamel dysplasia in children? A. Sulfamethoxazole B. Ethambutol C. Chloramphenicol D. Doxycycline E. Ciprofloxacin

D. Doxycycline

What is the preferred drug for the management of pure absence seizures? A. Phenobarbital B. Valproic acid C. Phenytoin D. Ethosuximide E. Lamotrigine

D. Ethosuximide

The transport of glucose that is facilitated by insulin is mediated by which of the following transporters? A. GLUT1 B. GLUT2 C. GLUT3 D. GLUT4 E. GLUT 5

D. GLUT4

Neuroleptic malignant syndrome which resembles malignant hyperthermia is more commonly seen with which of the following agents? A. Mesoridazine B. Promazine C. Loxapine D. Haloperidol E. Sertindole

D. Haloperidol

Which of the following inhalational anesthetics associated with potentially severe and Life-threatening hepatitis? A. Sevoflurane B Isoflurane C. Desflurane D. Halothane E. Nitrous Oxide

D. Halothane

Which of the following inhalational anesthetics sensitizes the myocardium to cathecolamines which can lead to ventricular arryhtmia when symphatomimetic drugs are co-currently administered? A. Sevoflurane B. Isoflurane C. Desflurane] D. Halothane E. Nitrous Oxide

D. Halothane

A 3 year old boy is diagnosed with primary tuberculosis infection. The appropriate anti-TB regimen for the child may include which of the following agents? I. Ethambutol II. Rifampicin III. Isoniazid A. I only B. II only C. I and III D. II and III E. I, II and III

D. II and III

Ginseng (Panax ginseng) is a popular additive in nutritional supplement due to its ability to improve physical and mental performance. Which subset/s of patients must be advised regarding the potential adverse effect of Ginseng on their disease or when taken concurrently with their medications? I. Patients with Bronchial asthma II. Patients taking warfarin III. Patients with hypertension A. I only B. II only C. I and III D. II and III E. I, II and III

D. II and III

Penicillins classified as ureido penicillins have extended spectrum of activity against gram-negative bacteria compared to the natural and other semisynthetic penicillins. Which of the following organism are the ureido penicillins effective against? I. Enterococci II. Klebsiella pneumoniae III. Pseudomonas aeruginosa A. I only B. II only C. I and III D. II and III E. I, II and III

D. II and III

Seizures as a manifestation of poisoning can be seen as a common component of the toxidrome of which of the following drugs? I. Atropine II. Methampethamine III. Tricyclic antidepressants A. I only B. II only C. I and III D. II and III E. I, II and III

D. II and III

Tardive dyskinesia is considered as the most important unwanted effect of antipsychotic agents. What is considered as the mechanism of development of this condition? I. Dopamine receptor block II. Relative cholinergic deficiency III. Dopamine receptor hypersensitivity A. I only B. II only C. I and III D. II and III E. I, II and III

D. II and III

What is the role of beta-blockers in the management of thyrotoxicosis? I. beta-blockers can inhibit the peroxidase-catalyzed reaction in the synthesis of thyroid hormones II. Beta-blockers control the sympathetic symptoms of hyperthyroidism III. Beta-blockers may inhibit the peripheral conversion of T4 to T3 A. I only B. II only C. I and III D. II and III E. I, II and III

D. II and III

Which of the following agents may be useful for eradicating a carrier state of Entamoeba histolytica? I. Metronidazole II. Diloxanide furoate III. Iodoquinol A. I only B. II only C. I and III D. II and III E. I, II and III

D. II and III

Which of the following are clinical use of Glucagon? I. Reverse the physical deformity associated with acromegaly II. Management of severe hypoglycemia III. Reverse the cardiac effects of beta blockers overdose A. I only B. II only C. I and III D. II and III E. I, II and III

D. II and III

Which of the following describes the mechanism of action of the thioamides for the management of hyperthyroidism? I. Block uptake of iodine by the follicular cells II. Block the peroxidase-catalyzed reactios III. Block coupling of iodotyrosines A. I only B. II only C. I and III D. II and III E. I, II and III

D. II and III

Which of the following drugs given for tuberculosis is/are bactericidal in activity at the usual dose given for TB? I. Ethambutol II. Rifampicin III. Isoniazid A. I only B. II only C. I and III D. II and III E. I, II and III

D. II and III

Which of the following drugs is recommended prophylaxis for travelers in malaria endemic areas with known chloroquineresistant strains of Plasmodium falciparum? I. Primaquine II. Mefloquine III. Atovaquone-proguanil A. I only B. II only C. I and III D. II and III E. I, II and III

D. II and III

Which of the following factors predisposed to increased intestinal lead absorption? I. Increased dietary Calcium II. Iron deficiency III. Ingestion in an empty stomach A. I only B. II only C. I and III D. II and III E. I, II and III

D. II and III

Which of the following interventions is/are applicable in cases of acute opioid overdose? I. Activated charcoal dose not have a role in the management of opoid poisoning II. Naloxone can be administered IV, IM, or SC III. If respiratory depression develops, 2mg of naloxone is administered and may be repeated as necessary A. I only B. II only C. I and III D. II and III E. I, II and III

D. II and III

Which of the following is/are clinical indication/s for the drug Clindmycin? I. Treatment of Severs gram-negative bacterial infection II. Treatment of severe anaerobic infections caused by bacteroides III. Prophylaxis of endocarditis in patients with valvular heart disease undergoing dental procedures A. I only B. II only C. I and III D. II and III E. I, II and III

D. II and III

Which of the following is/are important monitoring considerations in a patient receivimg Simvastatin? I. Baseline and follow-up measurement of serum creatinine II.Baseline and follow-up measurement of amino transferase activity III. Baseline and follow-up measurement of creatine kinase activity A. I only B. II only C. I and III D. II and III E. I, II and III

D. II and III

Which of the following patients may be expected to have an increased sensitivity to sedative-hypnotics leading to higher incidence of adverse effects or toxicity? I. Young women II. Patients with Cardiovascular disease III. Patients with respiratory disease A. I only B. II only C. I and III D. II and III E. I, II and III

D. II and III

Which of the following statements correctly describes the properties of Aminoglycosides? I. They are mainly bacteriostatic II. They are mainly effective against gram-negative aerobic bacteria III. They inhibit protein synthesis A. I only B. II only C. I and III D. II and III E. I, II and III

D. II and III

Which of the following statements in Sulfonamides is/are correct? I. Their effect is attributed to their ability to inhibit the enzyme dihydrofolate reductase II. As sine agents, they are bacteriostatic but are bactericidal when given with Trimethoprim III. They can cause hemolytic anemia if given to G6PD deficient individuals A. I only B. II only C. I and III D. II and III E. I, II and III

D. II and III

Which of the following third generation cephalosporins have activity against Pseudomonas aeruginosa? I. Ceftriaxone II. Ceftazidime III. Cefoperazone A. I only B. II only C. I and III D. II and III E. I, II and III

D. II and III

Immune insulin resistance is said to be due to production of low levels of what type of insulin antibodies? A. IgA B. IgD C. IgE D. IgG E. IgM

D. IgG

A 43 year old female who works in a hospital was admitted for tachycardia, palpitations and weight loss. Her T4 was elevated with suppresses TSH but her thyroid scan showed a normal-sized gland with normal iodine uptake. Which of the following should be suspected? A. Presence of Grave's disease B. Presence of solitary toxic nodule C. Iodine deficiency goiter D. Illicit intake of Levothyroxine E. Hypothyroidism

D. Illicit intake of Levothyroxine

Which of the following agents is most effective against nosocomial infection with Pseudomonas aeruginosa? A. Clarithromycin B. Ceftazidime C. Amikacin D. Imipenem E. Metronidazole

D. Imipenem

Which of the following mechanisms explains the toxicity of the chlorinated hydrocarbon insecticides like DDT? A. inhibit the ligand-gated sodium channels B. Inhibit the enzyme acetylcholinesterase C. Stimulate the GABA-A receptor D. Inhibit the inactivation of sodium channels E. Promote outward potassium transport

D. Inhibit the inactivation of sodium channels

What is the probable mechanism of action of Mebendazole, A broad spectrum anthelmintic agent for Ascariasis, Trichuriasis, and hookworm infection? A. Inhibition of the incorporation of PABA in to dehydropteroate B. Intensifies GABA-mediated transmission in peripheral nerves of nematodes C. Inhibition of cholinesterase action in parasites leading to paralysis of the worm D. Inhibition of the microtubule synthesis E. Increases permeability of the nematode cell membrane to calcium resulting to paralysis and death

D. Inhibition of the microtubule synthesis

Female patients with Pituitary tumor and manifesting with amenorrhea and galactorrhea are initially manage with Bromocriptine. What is the mechanism of Bromocriptine in this condition? A. It inhibits the action Dopamine B. It inhibits the action of Oxytocin C. It stimulates the release of Estrogen D. It inhibits the action of Prolactin E. It stimulates the release of FSH and LH

D. It inhibits the action of Prolactin

What is the use of Cilastatin? A. Effective agent against Pseudomonas aeruginosa B. It prevents tubular secretion of Imipenem C. It prevents glomerula filtration of Imipenem D. It inhibits the renal dehydropeptidase which metabolizes Imipenem E. It inhibits liver dehydropeptidase which metabolizes Imipenem

D. It inhibits the renal dehydropeptidase which metabolizes Imipenem

Which of the following anesthetic is associated with a Dissociative state characterized by catatonia, amnesia, and analgesia with or without actual loss of consciousness? A. Propofol B. Thiopental C. Midazolam D. Ketamine E. Nitrous Oxide

D. Ketamine

A 50 year old male patient with Type2 Diabetes mellitus is maintained for the last 7 days on subcutaneous Isophane Insulin at 20 units and 10 units pre-supper. For the last three days, his capillary blood glucose levels have been consistently showing the following values: 190 mg% (prebreakfast), 118mg% (2hr-post lunch), and 110 mg% (presupper). What recommendation will you make regarding his therapy? A. Increase his pre-breakfast insulin to 24 units while maintaining his pre-supper Insulin at 10 units B. Increase his pre-breakfast Insulin to 24 units and his pre-supper Insulin to 12 units C. Maintain his pre-breakfast and pre-supper Insulin doses give Insulin Lispro 4 units pre-breakfast D. Maintain his pre-breakfast insulin at 20 units while increase his pre-supper insulin to 12 units E. No change in the insulin doses as all values are within the acceptable range

D. Maintain his pre-breakfast insulin at 20 units while increase his pre-supper insulin to 12 units

What is the clinical use of the tetracycline drug Demeclocycline? A. Prophylaxis against plague B. Antiprotozoal agent C. Used to eradicate meningococcal carrier state D. Management of SIADH E. Management of Nephrogenic Diabetes Insipidus

D. Management of SIADH

Which of the following drugs is a Biguanide? A. Acetohexamide B. Rosiglitazone C. Nateglinide D. Metformin E. Acarbose

D. Metformin

Toxidrome consisting of CNS depression followed by severe metabolic acidosis, blindness with initial "snowstorm-like" visual disturbance, and coma is most consistent with which of the following toxicants? A. Nitrates B. Warfarin C. Ethanol D. Methanol E. Ethylene glycol

D. Methanol

What is the primary role of agents such as Tazobactam amd Sulbactam in antibacterial therapy? A. Increase in the oral bioavavilability of the Penicillins B. Effective against most gram-negative aerobic bacteria C. Effective against anaerobic bacteria including Clostridia D. Minimize destruction of the Penicillins by bacterial beta-lactamases E. Improve CNS penetrability of Penicillin

D. Minimize destruction of the Penicillins by bacterial beta-lactamases

Which of the following drugs combination provide a state called Neurolepanesthesia? A. Haloperidol + Thiamylal + Halothane B. Droperidol + Fentanyl + Nitrous Oxide C. Thioridazine + succinyl choline + propofol D. Nitroue Oxide + Droperidol +Fentanyl E. Halothane + Drperidol + fentanyl

D. Nitroue Oxide + Droperidol +Fentanyl

Which of the following antibiotic combinations is associated with clinically significant antimicrobial antagonism which has been reported to cause higher rate of mortality in patients given this combination compared to patients receiving any of the agent singly? A. Penicillins + Cephalosporin B. Penicillin + aminoglycoside C. Erythromycin + aminoglycoside D. Penicillin + Chloramphenicol E. Erythromycin + Doxycycline

D. Penicillin + Chloramphenicol

The absorption of most oral penicillins is impaired by food. An exception to this is: A. Ampicillin B. Amoxicillin C. Penicillin D. Penicillin G E. Oxacillin

D. Penicillin G

Which of the following agents is absolutely contraindicated in patients with disorder associated with porphyrin such as acute intermittent porphyria, vanegate porphyria, and symptomatic porphyria? A. Chlordiazepoxide B. Diazepam C. Zolpodem D. Phenobarbital E. Buspirone

D. Phenobarbital

Which of the following drugs used or seizure disorders is most commonly associated with cosmetic changes making the drug desirable to used among adolescents, school-age children, and women? A. Lamotrigine B. gabapentin C. Phenobarbital D. Phenytoin E. Valproic acid

D. Phenytoin

Which of the following TB drugs is given only during the intensive phase of therapy? A. Isoniazid B. Rifampicin C. Ethambutol D. Pyrazinamide E. Streptomycin

D. Pyrazinamide

Which of the following agents given for tuberculosis may provoke an attack of acute gouty arthritis? A. Isoniazid B. Rifampicin C. Ethambutol D. Pyrazinamide E. Streptomycin

D. Pyrazinamide

Which of the following agents reversibly binds to the 30S subunits of the bacterial ribosome blocking the binding of aminoacyltRNA to the acceptor site on the mRNA-ribosome complex? A. Aminoglycosides B. Chloramphenicol C. Linezolid D. Tetracyclines E. Macrolides

D. Tetracyclines

Dystoinic reactions such as an oculogyric crisis and pseudoparkinsonism such as tremors are side-effects most likely to be seen in patients taking which of the following medications? A. Thioridazine B. Chlorpromazine C. Loxapine D. Trifluphenazine E. Molindone

D. Trifluphenazine

When Phenytoin is given intravenously, what is the maximum rate by which drug can be administered? A. 10mg/min B. 20mg/min C. 30mg/min D. 40mg/min E. 50mg/min

E. 50mg/min

Chronic exposure with this metal has been linked with malignancies of the lung, skin, and the urinary bladder even at doses not high enough to cause either acute or chronic toxicities. Which toxic chronic exposures on the hand, important findings include a "raindrop" pattern of hyperpigmentation and hyperkeratosis of the palms and soloes,and a stocking-glove pattern of sensorimotor neuropathy? A. Lead B. Mercury C. Iron D. Copper E. Arsenic

E. Arsenic

The findings of congestive cardiomyopathy, pulmonary edema (cardiogenic and non-cardiogenic), pancytopenia with basophilic stippling of erythrocytes, and ascending sensorimotor peripheral neuropathy with acute exposure, and the appearance of Aldrich-mees lines months after the acute exposure are consistent with poisoning with which of the following metals? A. Lead B. Mercury C. Iron D. Copper E. Arsenic

E. Arsenic

Cardiotoxocity manifesting as a electromechanical dissociation, cardiovascular collapse and death, is a complication of therapy most commonly seen with which of the following local anesthetics? A. Mepivacaine B. Ropivacaine C. Lidocaine D. Procaine E. Bupivacaine

E. Bupivacaine

Which of the following is the preferred treatment of malaria caused by Plasmodium vivax or Plasmodium ovale? A. Clindamycin B. Doxycycline C. Fansidar D. Halofantrine E. Chloroquine

E. Chloroquine

.Which of the following agents is bactericidal in its activity? A. Clindamycin B. Erythromycin C. Chloramphenicol D. Doxycycline E. Ciprofloxacin

E. Ciprofloxacin

Which of the following agents is contraindicated in patients less than 18 years of age and pregnant patients due to its adverse effects on cartilage development? A. Sulfamethoxazole B. Ethambutol C. Chloramphenicol D. Doxycycline E. Ciprofloxacin

E. Ciprofloxacin

Dimercaptopropanesulfonic acid is also known as ______? A. Dimercaprol B. Succimer C. D-dimethylcysteine D. unithiol E. Deferoxamine

E. Deferoxamine

In developing a fetus as well as in a young child, what is considered as the most sensitive target organ for Lead's toxic effect? A. Active bone marrow B. Kidneys C. Immature reproductive organs D. Gastrointestinal tract E. Developing central nervous system

E. Developing central nervous system

Which of the following glucocorticosteroids is employed in the work-up of patients with Cushing's Syndrome to diagnose the most probable etiology of the condition? A. Hydrocortisone B. Prednisone C. Cortisone D. Fludprednisolone E. Dexamethasone

E. Dexamethasone

Hemodialysis as an intervention for poisoning is indicated for which of the following drugs? A. Morphine B. Digoxin C. Diazepam D. Propranolol E. Ethylene glycol

E. Ethylene glycol

Which of the following agents is expected to produce the effects and side effects comparable to Haloperidol? a. Mesoridazine B. Chlorprothixene C. Thiothixene D. Promazine E. Fluphenazine

E. Fluphenazine

What is the mechanism of action of Glucagon that makes the drug useful for the management of overdoses or poisoning with drugs that directly inhibit myocardial contractility? A. glucagon directly stimulates calcium release from the sarcoplasmic reticulum B. Glucagon directly binds to beta adrenergic receptors leading to an increase cAMP C. Glucagon binds to glucagons receptors that stimulates production of IP3 and cause an increase in intracellular calcium D. Glucagon binds to glucagons receptors leading to generation of cAMP E. Glucagon increases cAMP levels by inhibiting the enzyme phosphodiesterase

E. Glucagon increases cAMP levels by inhibiting the enzyme phosphodiesterase

A 20 year old male patient is admitted after ingesting 20 tablets of 300mg Isoniazid tablet. Which of the following statements is/are correct regarding the management of this patient? I. Administer a total of 6000mg pyridoxine in divided doses by slow IV II. Give activated charcoal slurry III. Given benzodiazepines to control seizures A. I only B. II only C. I and III D. II and III E. I, II and III

E. I, II and III

A patient is given physostigmine to manage poisoning with anti-muscarinic agent. Which of the following is/are possible complications of therapy with physostigmine? I. Cholinergic crisis II. Seizures III. Asystole A. I only B. II only C. I and III D. II and III E. I, II and III

E. I, II and III

In patients on glucocorticoid therapy, which of the following monitoring plans sould be initiated? I. Blood pressure monitoring II. Bone density monitoring III. Blood sugar monitoring A. I only B. II only C. I and III D. II and III E. I, II and III

E. I, II and III

In the management of Mercury Poisoning, which of the following is/are important considerations? I. dimercaprol is not used for elemental and organic mercury poisoning as the drug may redistribute mercury from tissues to brain II. Chelation from unithiol may diminished nephrotoxicity that may result from acute exposure to inorganic mercury III. In acute exposure, the efficacy of chelation therapy diminishes with time since exposure A. I only B. II only C. I and III D. II and III E. I, II and III

E. I, II and III

Milk may be used to dilute ingested toxicants. Which of the following ingested poisons may be manage initially by the administration of milk? I. Hypochlorite II. Corrosive acids III. Fluoride A. I only B. II only C. I and III D. II and III E. I, II and III

E. I, II and III

Which of the following agents are avoided in patients taking Tranylcypromine? I. Ephedrine II. Phenylpropanolamine III. Levodopa A. I only B. II only C. I and III D. II and III E. I, II and III

E. I, II and III

Which of the following agents can produce the so-called serotonin syndrome when given to a patient already on Phenelzine? I. Venlafaxine II. Buspirone III. Fluoxetine A. I only B. II only C. I and III D. II and III E. I, II and III

E. I, II and III

Which of the following agents is/are considered as botanical insecticide/s (derived from botanical sources)? I. Nicotine II. Rotenone III. Pyrethrum A. I only B. II only C. I and III D. II and III E. I, II and III

E. I, II and III

Which of the following agents may be considred as useful 2nd line agents for TB? I. Kanamycin II. Amikacin III. Ciprofloxacin A. I only B. II only C. I and III D. II and III E. I, II and III

E. I, II and III

Which of the following agents may be expected to have an additive CNS depressants effect with the sedative hypnotics like benzodiazepines when taken simultaneously? I.Opioid analgesics II. Phenothiazines III. Alcoholic beverages A. I only B. II only C. I and III D. II and III E. I, II and III

E. I, II and III

Which of the following agents may interfere with the absorption of levothyroxine that patient must be advised on the proper interval of intake with this/these agents? I. Sucralfate II. Ferrous sulfate III. Aluminum hydroxide A. I only B. II only C. I and III D. II and III E. I, II and III

E. I, II and III

Which of the following agents requires adjustment or may even have to be avoided in patients with hepatic impairment? I. Erythromycin II. Metronidazole III. Clindamycin A. I only B. II only C. I and III D. II and III E. I, II and III

E. I, II and III

Which of the following characteristics is consistent with the drug Propofol? I. Causes a marked reduction in blood pressure during induction II. Recovery from anesthesia is rapid III. Post-operative nausea and vomiting is minimal due to its anti-emetic effect A. I only B. II only C. I and III D. II and III E. I, II and III

E. I, II and III

Which of the following chelating agents may be useful in the different phases of management of inorganic lead poisoning? I. Calcium disodium edetate II. Succimer III. Dimercaprol A. I only B. II only C. I and III D. II and III E. I, II and III

E. I, II and III

Which of the following conditions enhance the toxicity of lithium? I. Use of thiazide diuretics II. Diarrhea and vomiting III. Excessive sweating A. I only B. II only C. I and III D. II and III E. I, II and III

E. I, II and III

Which of the following drugs can be bind to molecular components of the macromolecular complex of GABAA receptor -chloride channel present in neuronal membranes in the CNS? I. Diazepam II. Zolpidem III. Phenobarbital A. I only B. II only C. I and III D. II and III E. I, II and III

E. I, II and III

Which of the following findings is/are consistent with the poisoning of antipsychotic drugs? I. Hypotension II. Hypothermia III. Ventricular arrythmias A. I only B. II only C. I and III D. II and III E. I, II and III

E. I, II and III

Which of the following is consistent with the action of the drug Colestipol? I. Prevents reabsorption of bile acids in intestine II. Increases uptake of LDL and IDL with up-regulation of LDL receptors in the liver III. Causes an increase in conversion of cholesterol to bile acids A. I only B. II only C. I and III D. II and III E. I, II and III

E. I, II and III

Which of the following is/are advantage/s of the SSRI's compared to the rest of the drugs used for depression? I. Lesser lethality with overdose II. Generally no dose titration is required III. Minimal anticholinergic effects A. I only B. II only C. I and III D. II and III E. I, II and III

E. I, II and III

Which of the following is/are appropriate indications for oxytocin? I. Control of postpartum uterine bleeding II. Stimulate initial milk production III. Induction of labor A. I only B. II only C. I and III D. II and III E. I, II and III

E. I, II and III

Which of the following is/are correct indications for the use of Dimercaprol? I. Single-agent treatment of acute poisoning with inorganic or elemental mercury II. Single-agent treatment of acute arsenic poisoning III. Combination treatment with edentate calcium disodium for severe lead poisoning A. I only B. II only C. I and III D. II and III E. I, II and III

E. I, II and III

Which of the following may be considered as indication for the streptogramin combination Quinupristin-Dalfopristin? I. Treatment of MRSA II. Treatment of Multi-Drug resistant strains of Streptococci III. Treatment of Penicillin-resistant strain of Streptococcus pneumonia A. I only B. II only C. I and III D. II and III E. I, II and III

E. I, II and III

Which of the following mechanisms explain/s the action of Phenytoin? I. The drug alters conductance of sodium, potassium, and calcium II. At high concentrations, it also inhibits the release of Serotonin and Norepinephrine III. At therapeutic concentrations, Phenytoin inhibits the generation of repetitive action potentials A. I only B. II only C. I and III D. II and III E. I, II and III

E. I, II and III

Which of the following organism is/are susceptible to Penicillin G, making the drug the drug of choice in the treatment of the infection caused by this/these organism? I. Treponema pallidum II.Pneumococcus III. Beta-hemolytic streptococci A. I only B. II only C. I and III D. II and III E. I, II and III

E. I, II and III

Which of the following statements correctly describes the thyroid hormone replacement therapy? I. Infants and children with congenital hypothyroidism require higher dose per kilogram body weight than adults II. Steady state levels of thyroxine takes about 6-8 weeks to achieve after initiating therapy III. Older adults and those with long-standing disease must be started on lower than usual dose of levothyroxine A. I only B. II only C. I and III D. II and III E. I, II and III

E. I, II and III

Which of the following statements regarding carbon monoxide poisoning is/are correct? I. The brain and the heart are the most affected organ II. Forms carboxyhemoglobin which cannot transport oxygen III. Smokingmales have a higher carboxyhemoglobin level A. I only B. II only C. I and III D. II and III E. I, II and III

E. I, II and III

Delayed neurotoxicity associated with exposure to organophosphate characterized by polyneuropathy, paralysis and axonal degeneration has been attributed to what effect of these toxicants? A. Inhibition of the plasma butyrylcholinesterase B. Inhibition of release of Acetylcholine C. Inhibition of the RBC acetylcholinesterase D. Direct inhibition of myelin sheath function E. Inhibition of the neuropathy target esterase

E. Inhibition of the neuropathy target esterase

The mechanism of action of Clindamycin is similar to the mechanism of action of which of the following agents? A. Aminoglycosides B. Chloramphenicol C. Linezolid D. Tetracyclines E. Macrolides

E. Macrolides

Which of the following agents primarily work by inhibiting aminoacyl translocation and blocking the formation of initiation complexes? A. Aminoglycosides B. Chloramphenicol C. Linezolid D. Tetracyclines E. Macrolides

E. Macrolides

Weight gain is expected complication of therapy with all the clinically useful antipsychotics. One of the drugs listed below, however does not produce this side effect. What is this drug? A. Thioridazine B. Chlorpromazine C. Loxapine D. Trifluphenazine E. Molindone

E. Molindone

What is considered as the most common cause of air pollution? A. Sulfur oxides B. Nitrogen oxides C. Hydrocarbons D. Carbon monoxide E. Ozone

E. Ozone

Which of the following Local anesthetics can predisposed to the development of methemoglobinemia when given in large doses during regional anesthesia? A. Procaine B. Lidocaine C. Cocaine D. Tetracaine E. Prilocaine

E. Prilocaine

What is the primary advantage of Insulin Lispro compared to regular Insulin? A. Longer duration of action reaching up to 6 hours B. No associated peak levels C. It is less immunogenic D. Less likelihood to cause hypoglycemic episodes E. Rapid onset of action in 5 to 15 minutes

E. Rapid onset of action in 5 to 15 minutes

Which of the following drugs is involved in regulating the genes involved in lipid and glucose metabolism and adiposite differentiation by acting as ligand of the PPAR-gamma, thus useful for the managing Insulin resistance? A. Metformin B. Repaglinitide C. Acarbose D. Tolazamide E. Rosiglitazone

E. Rosiglitazone

Which of the following Local anesthetics is hydrolyzed to Para-amino benzoic acid? A. Cocaine B. Isobucaine C. Lidocaine D. Bupivacaine E. Tetracaine

E. Tetracaine

What is responsible for the very short duration and very rapid onset of effect of the so-called Ultra short acting barbiturates? A. The presence of a Sulfur group B. The presence of Methoxy group C. The presence of substituted barbital structure D. The high lipid solubility of these agents E. The increased in the number of polar substituents

E. The increased in the number of polar substituents

Parathion has a toxicity rating of 6. What are the probable human lethal dosages with this rating? A. 5,000-50,000mg/kg body weight B. 500- 5,000mg/kg body wt C. 50- 500mg/kg body wt D. 5-50mg/kg body wt E. not more than 5 mg/kg body wt

E. not more than 5 mg/kg body wt

Which of the following agents requires weekly WBC monitoring within the first 3 months of treatment due to a risk of causing leukopenia? A. Cilostazol B. Dipyridamole C. abciximab D. Clopidogrel E. Ticlodipine

E.Ticlodipine

What is the primary mechanism involved in the utility of opioids in the management of acute pulmonary edema? a. Induction of osmotic diiuresis of edema fluuids b. Peripheral vasodilation reducing venous return c. Decreased cardiac inotropism d. Improved mobilization of edema fluid by osmosis e. Increased glumerolar filtration rate

Peripheral vasodilation reducing venous return

At what dose should Dopamine be given as IV infusion to produce a predominantly dopaminergic effect? a. 0.5 ug/kg/min b. 5-10ug/kg/min c. 10-20ug/kg/min d. 20-50ug/kg/min e. >50ug/kg/min

a. 0.5 ug/kg/min

During the conduct of ACLS, what is the recommended dosing ofor Epinephrine in adults? a. 1mg every 3-5 minutes b. 3mg every 3-5 minutes c. Escalating dose, staring at 1mg then increasing by 1mg every 3-5 minutes d. 10ml of a 1:1000 solution every 3-5 minutes

a. 1mg every 3-5 minutes

Which of the following non-adrenergic, non-cholinergic neurotransmitters is involved in the action of the enteric nervous system? a. ATP b. Dopamine c. GABA d. Glutamate e. Glycine

a. ATP

What is the neurotransmitter mainly released by postganglionic parasympathetic fibers? a. Acetylcholine b. Norepinephrine c. Epinephrine d. 5-Hydroxyryptamine e. Dopamine

a. Acetylcholine

What neurotransmitter is released by preganglionic fibers as a response to sympathetic stimulation? a. Acetylcholine b. Norepinephrine c. Epinephrine d. Serotonin e. Dopamine

a. Acetylcholine

What is the mechanism of action of regular heparin? a. Activates antithrombin III which in turn inactivates a number of activated clotting factors b. Activation of plasminogen to plasmin leading to destruction of fibrin and fibrinogen c. Inhibits the reductive activation of vitamin K epoxide to the hydroquinone form d. Inhibits activvation of prothrombin to thrombin e. Mimics the action of epsilon-aminocaproic acid

a. Activates antithrombin III which in turn inactivates a number of activated clotting factors

Which of the following mechanisms explains the action of the drug Sodium Nitropusside? a. Activation of guanylyl cyclase with increase of cGMP b. Release of N20 with subsequent increase in cAMP c. Inhibition of the secretion of renin d. Stimulation of A1 receptors e. Increase conductance of outward potassium channels

a. Activation of guanylyl cyclase with increase of cGMP

What class of H1-antihistamines include drugs that sre used as components of OTC "cold" medications a. Alkylamines b. Phenothiazine c. Piperazine d. Ethanolamine e. Ethylaminediamine

a. Alkylamines

Which of the following drugs used for gout produces its effect by inhibiting the enzyme xanthine oxide a. Allopurinol b. Colchicine c. Indomethacin d. Probenecid e. Sulfinpyrazone

a. Allopurinol

Charcoal-broiled foods, cruciferous vegetables and omeprazole induce the metabolism of drugs like tomoxifen, acetaminophen, theophylline and warfarin. This effect is due to induction of which CYP isoform? a. CYP1A2 b. CYP2A6 c. CYP2C9 d. CYP2D6 e. CYP4A4

a. CYP1A2

Drugs that activate the phospholipase C-inositol phosphate system such as alpha-adrenergic agonists like phenylephrine primarily cause an increase in the intracellular levels of which ion? a. Calcium b. Magnesium c. Sodium d. Potassium e. Chloride

a. Calcium

A 47 year old female diabetic is maintained on Tolbutamide for control of her hyperglycemia. Her fasting blood sugar is maintained at 102mg%. Three days prior to admission, she self-medicated with Indomethacin due to a joint pain. She was admitted at the emergency room unconscious and difficult to arouse. What laboratory finding may be consistent with the history of the patient? a. Capillary blood glucose of 20mg/dL b. Capillary blood glucose of 400mg/dL c. ALT level that is 3x elevated d. ALT level that is 6x elevated e. Platelet level of 100,000/ul

a. Capillary blood glucose of 20mg/dL

Which of the following drugs or conditions can increase the activity of warfarin by affecting the drug's absorption, distribution, and elimination? a. Cimetidine b. Rifampicin change to vegetarian diet c. The presence of hepatic disease d. Concurrent use of heparin

a. Cimetidine

Tramadol is weak mu-receptor agonist that does not require as S2 license for dispensing. It is a synthetic analogue ofwhich of the following opiates/opioids? a. Codeine b. Thebaine c. Morphine d. Meperidine e. Diphenoxylate

a. Codeine

In a patient with hepatic encelopathy, which of the following agents may lead tosevere drug toxicity even when given at usual doses? a. Diazepam b. Oral Neomycin c. Lactulose d. Propanolol e. IV Gentamicin

a. Diazepam

Which of the following drugs inhibit enzyme carbonic anhydrase? a. Dichlorphenamide b. Tolazamide c. Astemizole d. Selegiline e. Donepezil

a. Dichlorphenamide

Which of the following H1-antihistamines can block sodium channels in excitable membranes bringing about local anesthetic effect? a. Diphenhydramine b. Meclizine c. Doxylamine d. Pyrilamine e. Cyproheptadine

a. Diphenhydramine

Which of the following H1-antihistamines has significant anti-cholinergic effect to be useful for the management of dytonia and pseudoparkinsonism associated with the use of neuroleptic agents? a. Diphenhydramine b. Brompheniramine c. Promethazine d. Cyproheptadine e. Cyclizine

a. Diphenhydramine

Which of the following drugs primarily stimulates beta-1 receptors? a. Dobutammine b. Ritodrine c. Yohimbine d. Phenoxybenzamine e. Oxymetazoline

a. Dobutammine

Which of the following agents must be classified as a low molecular weight heparin? a. Enoxaparin b. Regular heparin c. Warfarin d. Hirudin e. Phenindione

a. Enoxaparin

Which of the following agents maybe associated with ototoxicity which may have increased risk of occuring when patient also recieve aminoglycoside antibiotic? a. Furosemide b. Chlorthalidone c. Dorzolamide d. Amiloride e. Spironolactone

a. Furosemide

Which of the following agents is a strong full agonist of opioid receptors a. Hydromorphone b. Hydrocodone c. Codeine d. Propoxyphene e. Pentazocine

a. Hydromorphone

A patient complains of muscle weakness after several doses of Salbutamol nebulization. This condition can be attributed to _______________? a. Hypokalemia induced by Salbutamol therapy b. Hypocalcemia induced by Salbutamol therapy c. Metabolic alkalosis due to bronchial asthma d. Decreased firing of neuromuscular junction fibers as a direct effect of Salbutamol e. Respiratory alkalosis due to bronchial asthma

a. Hypokalemia induced by Salbutamol therapy

In which of the following conditions are beta-blockers found to be useful? I. Hypothyroidism II. Raynaud's Phenomenon III. Initial therapy in Pheochromocytoma a. I only b. I and II c. III only d. I and III e. I, II, and III

a. I only

What is/are the clinical use/s of Edorphonium (Tensilon)? I. Reverse the effect of neuromuscular blockers after surgery II. Treatment of overdose of atropine III. Management of organophosphate poisoning a. I only b. I and II c. III only d. I and III e. I, II, and III

a. I only

Which of the following drugs or drug combinations can reduce both the cardiac preload and afterload in CHF? I. Hydralazine + Nitrovasodilators II. Beta-blockers + Loop diuretics III. Loop diuretics + Digitalis a. I only b. II only c. I and III d. II and III e. I, II, and III

a. I only

Which of the following effects can be attributed to histamine binding to its H1 receptor? I. Extravascular smooth muscle contraction II. Vascular smooth muscle contraction III. Gastric acid secretion a. I only b. II only c. I and III d. II and III e. I, II, and III

a. I only

Which of the following intervention may be appropriate in the management of severe cases of digitalis intoxicationsuch as in children and in suicidal overdose? I. Administration of digoxin immune fab II. Aadministration of KCl III. Administration of MgSO4 a. I only b. II only c. I and III d. II and III e. I, II, and III

a. I only

Which of the following statements describe the use of anti-cholinergic agents in COPD and bronchial asthma? I. They can be safely given at high doses by inhalations to saturate the muscarinic receptors II. Available agents are tertiary ammonium compunds III. They are more effective than beta-2 agonists in bronchial asthma but less effective in COPD a. I only b. II only c. I and III d. II and III e. I, II, and III

a. I only

Which of the following statements regarding the development of tolerance to opioids is/are correct? I. Develops in 2-3 weeks of continuous use especially when large doses are used over short intervals II. Gross-tolerance develops among predominantly kappa-receptor full agonists III. Partial agonists induce cross-tolerance with strong agonists and antagonists a. I only b. II only c. I and III d. II and III e. I, II, and III

a. I only

The effect of muscarinic receptor activation in tissues like the exocrine glands and the extravascular smooth muscles have been linked with the generation of which second messenger molecule? a. IP3 b. cAMP c. cGMP d. ATP e. Adenosine

a. IP3

The efficacy of ACE-inhibitors in the management of hypertension may be reduced in the presence of certain drugs such as ___________? a. Ibuprofen b. Felodipine c. Indapamide d. Eprenolone e. Hydralazine

a. Ibuprofen

Which of the following statements on pharmacokinetic parameters must be considered for patients taking Digoxin? a. In about 10% of patients, use of antibiotics can lead to increased oral bioavailability b. Renal clearance of the drug is inversely proprtional to the creatinine clearance c. Half-life is long, at about 168 hours due in part to extensive enterohepatic recirculation d. Hepatic metabolism accounts for more than 80% of the elimination of the drug e. It is primarily distributed within the intravascular space due to its high protein binding

a. In about 10% of patients, use of antibiotics can lead to increased oral bioavailability

The decrease in arterial tone due to vasorelaxant action of sodium nitropusside is associated with a. Increase in smooth muscle cGMP levels b. Increase in smooth muscle cAMP levels c. A decrease in calcium entry through L-type channels d. A local anethetic effect on smooth muscle cells e. Stimulation of inward K-channel

a. Increase in smooth muscle cGMP levels b. Increase in smooth muscle cAMP levels

Which of the following drugs classified as NSAIDs inhibit/s the COX-1 more than the COX-2 isozymes? a. Indomethacin b. Ibuprofen c. Mefenamic acid d. Naproxen e. Celecoxib

a. Indomethacin

What is the mechanism of action of the diydropyridine calcium channel blockers? a. Inhibit the L-type calcium channels in the arteriolar smooth muscles b. Inhibit the L-type calcium channels in the anteriolar and venous smooth muscles c. Equally inhibit the L-type calcium channels in the vascular and cardiac tissues d. Inhibit the S-type calcium channels in the anteriolar smooth muscles e. Inhibit the S-type calcium channels in the anteriolar and venous smooth muscles

a. Inhibit the L-type calcium channels in the arteriolar smooth muscles

Stimulation of Alpha-1 receptors leads to the formation of important secondary messengers that promote calcium release and activation of protien kinase C. These secondary messengers responsible for the effects of alpha-1 stimulation include which of the following? a. Inositol triphosphate b. Cyclic adenosine monophophate c. Adenosine triphosphate d. Cyclic guanosine monophosphate e. Adenosine diphosphate

a. Inositol triphosphate

Which of the following inhilational anesthetics produces greatest augmentation of the effect of neuromuscular blockers? a. Isoflurane b. Sevoflurane c. Halothane d. Desflurane e. Nitrous oxide

a. Isoflurane

Which of the availabe nitrovasodilators has a bioavailabilty of about 100%? a. Isosorbide mononitrate b. Isosorbide dinitrate c. Amyl nitrite d. Glyceryl trinitrate e. None aqs all undergo significant first pass effect when given orally

a. Isosorbide mononitrate

In comparison to other aantimuscarinic agents, Scopolamineis expected to exert significant CNS effects due to: a. Its being a tertiary amine b. Its being a quarternary amine c. The presence of a benzene ring structure d. The tropic acid structure its isomeric structure

a. Its being a tertiary amine

Which of the following is an opioid receptor that is responsible for inducing analgesia most especially among women? a. Kappa b. Delta c. Alpha d. Epsilon e. Mu

a. Kappa

Rebound tachycardia and hypertension are expected complications of abrupt withdrawal of beta-blocker therapy. Which of the followingagents has the least likelihood of causing rebound effects when withdrawn? a. Labetalol b. Propanolol c. Pindolol d. Metropolol e. Timolol

a. Labetalol

What is the primary advantage od drugs like Celecoxib and etericoxib? a. Less incidence of gastric irritation or ulceration b. Less incidence of nephrotoxicity c. Less incidence of hematologic toxicities d. Minimal risk of causing worsening hypertension e. Less likelihood of inhibiting platelet aggregation

a. Less incidence of gastric irritation or ulceration

What is the primary advantage of Angiotensin II receptor blockers over the ACE-Inhibitors? a. Lesser incidence of coughs b. Absence of risk interstitial nephritis c. Safe for pregnant patients with hypertension d. Faster onset of action e. Lesser incidence of hyperkalemia

a. Lesser incidence of coughs

What is the primary indication of the prostaglandin analogue Latanoprost? a. Management of glaucoma b. Smooth muscle relaxant in erectile dysfunction c. Abortifacient d. Inhibition of platelet aggregation e. Vasoconstrictor in episodes of hypertension

a. Management of glaucoma

The action of this drug is dependent on a colligative property a. Mannitol b. Cocaine c. Reserpine d. Furosemide e. Losartan

a. Mannitol

Which of the folling agents is primarily indicated for the management of increased intracranial pressure a. Mannitol b. Acetazolamide c. Furosemide d. Chlorthalidone e. Bumetanide

a. Mannitol

The mechanism of action of the drug clonidine most closely resembles that of ______________? a. Methyldopa b. Propanolol c. Carvedilol d. Guanethidine e. Reserpine

a. Methyldopa

A new drug is currently being developed whose mechanism of action is stimulation of the beta-3 receptors. In which condition will this drug have most likely usefulness? a. Obesity b. Bronchial asthma c. Hypertension d. Shock e. Acute renal failure

a. Obesity

Which of the following drug is currently considered as the preferred drug to prevent erosive gastritis in patient at risk of developing this side effect when NSAIDs have to be given to the patient? a. Omeprazole b. Ranitidine c. Sucralfate d. Telenzepine e. Misoprostol

a. Omeprazole

What is considered as dominant phase I metaabolic reaction? a. Oxidation b. Reduction c. Hydrolysis d. Glucuronidation e. Acetylation

a. Oxidation

Which of the following statements correctly explains the primary mechanism of relief of angina pectoris with the use of nitrovasodilators when given at usual doses? a. Peripheral dilation of the veins leading to reduction in cardiac preload and myocardial oxygen demand b. Vasodilation of epicardial coronary arteries leading to improved oxygen delivery to myocardial tissues c. Increase in diastolic perfusion time leading to improved perfusion to ischemic myocardium d. Decrease total peripheral resistance with arteriolar vasodilation e. Decrease in myocardial contractility leading to reduction in myocardial oxygen demand

a. Peripheral dilation of the veins leading to reduction in cardiac preload and myocardial oxygen demand

What is the mechanism of action of Zafirlukast and Montelukast? a. Pharmacologic antagonism of Luekotriene D4 action b. Pharmacologic antagonism of Leukotriene C4 action c. Inhibition of the enzyme 5-lipoxygenase d. Inhibition of citokine release by macrophages e. Inhibition of Histamine-1 receptors

a. Pharmacologic antagonism of Luekotriene D4 action

In the management of Pparoxysmal hypertensionin pheochromocytoma, which of the following drugs is the most appropriate initial therapy? a. Phentolamine b. Metoprolol c. Guanfacine d. Reserpine e. Clonidine

a. Phentolamine

The medium dose, or the dose of the drug that produces half of the maximum achievable response is called a. Potency b. LD50 c. Therapeutic index d. Ceiling dose e. Binding constant

a. Potency

Which of the following anti-hypertensive agents is the most appropriate for elderly male patients with benign prostatic hyperplasia? a. Prazosin b. Amlodipine c. Metoprolol d. HCTZ e. Enalapril

a. Prazosin

Which of the following is classified as a thrombolytic agent? a. Reteplase b. Warfarin c. Argatroban d. Bivalirudin e. Lepirudin

a. Reteplase

What is the most frequent side effect of methyldopa which can be seen particularly at the onset of treatment? a. Sedation b. Depression c. Extrapyramidal signs d. (+) coomb's test e. Galactorrhea

a. Sedation

The action of the drug Eplerenone most closely resembles which of the following agents? a. Spironolactone b. Amiloride c. Triamterence d. Metolazone e. Quinethazone

a. Spironolactone

Which of the following features cahracterize nicotinic, GABA-A, and Glutamate receptors? a. Thay control the movements of ions and out of the cell b. Their action is mediated bysecondary messengers c. They all have excitatory effect on the cell membrane d. They are located on the cytoplasm

a. Thay control the movements of ions and out of the cell

What is the main reason why atropine is always given with diphenoxylate (lomotil)? a. The anticholinergic after-effects with Atropine discourages abuse of Diphenoxylate b. The anti-motility effect of Atroppine augments the constipating effect of Diphenoxylate c. Atropine minimizes the respiratory depression expected with Diphenoxylate d. Atropine reduces the absorption and systemic toxicity of Diphenoxylate e. Atropine inhibits the hepatic metabolism of diphenoxylate effectively reducing the dose requuired for diarrhea

a. The anticholinergic after-effects with Atropine discourages abuse of Diphenoxylate

When warfarin is used as anticoagulant, the achievement of a maximal hypothrombinemic effect is expected to be delayed. What is the primary reason for this delay in warfarin effect? a. The presence of preformed clotting factors in the plasma which have biiologic half-lives 6 to 60 hours b. The high degree of protein binding which dela entry of the drug into its biologic site of action c. The delay of activation of precursor clotting factors due to high levels of activation-inhibitors d. The ability of warfarin to induce its own metabolism which delays achievement of therapeutic levels by days e. The ability of warfarin to interfere with the assay done to measure its activity

a. The presence of preformed clotting factors in the plasma which have biiologic half-lives 6 to 60 hours

When regular heparin is used subcutaneously, which of the followinh statements is correct? a. There is no need to monitor activity in most cases b. Monitoring of activity must be done on a weekly instead of daily basis c. Monitoring must be done using aPTT d. Monitoring of activity must be done regularly using the PT value e. Monitoring of activity must be done using the platelet count

a. There is no need to monitor activity in most cases

A patient has been assessed to have poor Debrisoquin metabolizing capability. Which of the following drugs will have an increased likelihood of causing toxicity in this patient? a. Thioridazine b. Amiodarone c. Acetaminophen d. Warfarin e. Theophylline

a. Thioridazine

This side effect of nitrovasodilators is explained in part by the depletion of sulfhydryl moieties in vascular smooth muscles with continuous use of the drugs, and may be partly reversed bysulfhydryl-regenerating compunds. a. Tolerance b. Increased intracranial pressure c. Reflex tachycardia d. Methemoglobenia e. carcinogenicity

a. Tolerance

A number of the NSAIDs cannot be given to patients with Gouty Arthritis as these may precipitate an acute attack of gout or induce uric acid stone formation. Which of the following NSAIDs should no be given to patient with gout? a. Tolmetin b. Ibuprofen c. Indomethacin d. Diclofenac e. Mefenamic acid

a. Tolmetin

Bethanecol is clinically used for which of the following conditions? a. Urinary retention in neurogenic bladder b. Intestinal obstruction c. Chronic glaucoma d. Supraventicular tachycardia e. Congestive heart failure

a. Urinary retention in neurogenic bladder

Which of the following substances may be absorbed significantly by pinocytosis a. Vitamin K b. Cyanocobalamin c. Cardiac glycosides d. Quarternary ammonium compounds e. Inorganic and organic electrolytes

a. Vitamin K

Which of the following features charcterizes the autonomic nervous system a. Single neuron transmission b. 0 c. Effects are under volitional control d. Conveys outpus from the CNS to the skeletal muscles e. Terminal neurotransmitter is mainly Serotonin

b. 0

What is the main final metabolite of Norepinephrine and Epinephrine which when abnormally elevated is often used as a marker for the disease Pheochromocytoma? a. Dihydroxymandelic acid b. 3-Methoxy-4 hydromandelic acid c. 3-Methoxy-4 hydroxyphenylglycol d. 3,4-Dihydroxyphenylglycol e. Normetanephrine

b. 3-Methoxy-4 hydromandelic acid

A25-years old male ingested an unidentified number of Aspirin tablets. He presents with the following findings: Responsive to verbal stimulation, BP=110/60, Body Temperature=39.5'C, ABGs: pH=6.9, HCO3=7meq/L, pCO2 -29 mmHg; dry oral mucosa, absent axillary sweating. What serum salicylate level is consistent with these findings? a. 50-80mg/dL b. 80-110mg/dL c. 110-160mg/dL d. 160-190mg/dL e. >190mg/dL

b. 80-110mg/dL

Which of the following agents classified as nitrovasodilators is useful in the management of cyanide poisoning? a. Sodium nitrite b. Amyl nitrite c. Sodium thiosulfate d. Nitroglycerin e. Isosorbide mononitrate

b. Amyl nitrite

In compartmental analysis of serum drug concentration versus time plots, which of the following findings confirm a one compartment model of drug behavior? a. An AUC above the explorated line that is less than 10% of the total AUC b. An AUC above the explorated line that is less than 5% of the total AUC c. Slope of the last 3 terminal points differing by more than 10% for the slope of the first 3 terminal points d. Slope of the 3 terminal points differing by less than 20% from the slope of the 3 terminal points e. Cmax that is above the line explorated from the terminal points

b. An AUC above the explorated line that is less than 5% of the total AUC

What is the clinical use of the drugs Ondansetron and Granisetron? a. Appetite suppressants b. Anti-emetic c. Treatment of acute migraine attack d. Migraine prophylaxis e. Motility enhancing agent

b. Anti-emetic

Which of the following equations describe the factors that contribute to blood pressure? a. BP=SV/SVR b. BP=COxSVR c. BP=SVxHR d. BP=CO/SVR e. BP=SVxSVR

b. BP=COxSVR

Among diabetic patients using insulin or oral hypoglycemic agents (OHAs) why are beta-blockers are used with special caution if not totally avoided? a. Beta-blockers can induce hyperglycemia b. Beta-blockers can mask hypoglemic symptoms c. Beta-blockers can attenuate effect of insulin and OHAs d. Beta-blockers can induce insulin or OHA failure e. Beta-blockers can increase the risk of renal failure

b. Beta-blockers can mask hypoglemic symptoms

For drugs that are given constant rate, the time to reach steady rate state concentration dependent on a. Rate of administration b. Biologic half-life c. Volume distribution d. Peak plasma concentration achieved e. Amount of the drug in the body

b. Biologic half-life

Which of the following correctly describes the action of cromolyn sodium? a. Induces relaxation of bronchial smooth muscles b. Blocks the degranulation of mast cells c. Inhibits release of cytokines by inflammatory cells d. Inhibits binding of IgE to mast cell membrane e. Inhibits degradation of cGMP

b. Blocks the degranulation of mast cells c. Inhibits release of cytokines by inflammatory cells

Which of the following drugs used for gout produces its effect by inhibiting the synthesis of microtubules? a. Allopurinol b. Colchicine c. Indomethacin d. Probenecid e. Sulfinpyrazone

b. Colchicine

In the biosynthesis of Norepinephrine, what step is considered rate-limiting? a. Active uptake of Tyrosine b. Conversion of tyrosine to dihydroxyphenylalanine c. Conversion of DOPA to dopamine d. Uptake of Dopamine in presynaptic storage vesicles e. Conversion of Norepinephrine to Epinephrine

b. Conversion of tyrosine to dihydroxyphenylalanine

Which of the following H1-antihistamines has significant blocking effects on 5-HT receptors? a. Promethazine b. Cyproheptadine c. Pyrilamine d. Carbinoxamine e. Cetirizine

b. Cyproheptadine

The PT-INR of a patient with Warfarin has been maintained at 2.5 for the last 3 months. 1 week earlier, the patient was given Phenobarbital. What can be expected with the PT-INR of the patient? a. Increased PT-INR b. Decreased PT-INR c. Maintained PT-INR

b. Decreased PT-INR

Which of the following mechanism may explain the development of tolerance to drugs? a. Increase in the quantity of mediators b. Decreased metabolic degradation of the active drug c. Decreased excretion or extrusion of drugs from cells d. Downregulation of receptors e. Inactivation of counterregulatory physiologic response

b. Decreased metabolic degradation of the active drug

Which of the following statements most closelydescribes the primary mechanism of action of the nitrovasodilator? a. Denitrification by glutathione S-transferase leads to release of nitrite ion which is then converted to nitric oxide, a molecule that activates adenylyl cyclase leading to increase cAMP b. Denitrification by glutathione S-transferase leads to release of nitrite ion which is then converted to nitric oxide, a molecule that activates guanylyl cyclase leading to increase cGMP c. Nitrate ion released upon denitrification directly stimulas outward K-channels leading to hyperpolarization of the vascular smooth muscles d. Nitrate ion released upon denitrification directly stimulas outward K-channels leading to hyperpolarization of the vascular smooth muscles e. Nitrate ion released upon denitrification stimulates the production of prostacylin leading to vasodilatio

b. Denitrification by glutathione S-transferase leads to release of nitrite ion which is then converted to nitric oxide, a molecule that activates guanylyl cyclase leading to increase cGMP

Which of the following agents is used primarily for its sedating property as the active ingredient of an OTC sleeping aid called Unisom? a. Carbinoxamine b. Doxylamine c. Hydoxyzine d. Meclizine e. Tripelenamine

b. Doxylamine

Cutaneous necrosis which can develop during the frist few weeks of therapy is expected with warfarin due to which of the following mechanisms? a. Idiosyncratic and paradoxical increase in the rate of synthesis of a number of clotting factors b. Earlier decline in the circulating levels of natural anticoagulants causing predominance of procoagulant clotting factors c. Intrinsic thrombogenic property of warfarin molecules d. Direct reduction in tissue perfusion as a consequence of the allergic property of warfarin e. Hypotensive effective of warfarin which reditributes blood flow to critical sites in the body

b. Earlier decline in the circulating levels of natural anticoagulants causing predominance of procoagulant clotting factors

Which of the following beta-blockers has the shortest duration of action (half-life = 10 minutes) making is useful for administration as continuous IV infusion with rapid achievemnet of a steady state concentration a. Metoprolol b. Esmolol c. Atenolol d. Propanolol e. Pindolol

b. Esmolol

Which of the following drugs is a sodium channel blockers which has no significant effect on the duration of the action potential and dissociate from the channel with slow kinetics? a. Lidocaine b. Flecainide c. Quinidine d. Bretylium e. Sotalol

b. Flecainide

. Oxynutinin, Dicyclomine, and Glycopyrrolate are clinically useful for which of the following conditions? a. Cholinomimetic agent overdose b. Gastrointestinal hypermotility and urinary incontinence c. Angina due to coronary artery vasospasm d. Motion sickness and Parkinsonism e. COPD and bronchial asthma

b. Gastrointestinal hypermotility and urinary incontinence

Which of the following is an expected respose to alpha-1 adrenoceptor stimulation? a. Uterine smooth muscle relaxation b. Gastrointestinal sphincter constriction c. Ciliary muscle relaxation d. Gastrointestinal wall smoothe muscle contraction e. Platelet aggregation

b. Gastrointestinal sphincter constriction

Which of the following agents is most useful for the management of nephrolithiasisndue to idiopathic hypercalciuria? a. Ethacrynic acid b. HCTZ c. Dorzolamide d. Vasopressin e. Triamterene

b. HCTZ

What is the most important toxicity associated with pyrazolone derivatives like Phenylbutazone which necessitated the withdrawal of a number of these drugs from the market? a. Hepatoxicity b. Hematologic toxicities c. Nephrotoxicity d. Gastric ulceration e. Stevens-johnson syndrome

b. Hematologic toxicities

In which of the following clincal conditions are the agonists of the alpha-1 adrenoceptors found to be useful? I. Nasal congestion II. Hypotension III. Sympomatic bradycardia a. I only b. I and II c. III only d. I and III e. I, II, and III

b. I and II

In which subset/s of patients are beta-blockers used with caution if not avoided? I. Patients with bronchospastic disease like COPD and bronchial asthma II. Patients with peripheral vascular disease especially vasospastic type III. Patients with recent myocardial infarction a. I only b. I and II c. III only d. I and III e. I, II, and III

b. I and II

The prerequisite that drugs must be present in aqueous solution for them to be absorbed is true if the drugs are absorbed by which mechanism? I. Passive transport II. Carrier-mediated transport III. Pinocytosis a. I only b. I and II c. III only d. I and III e. I, II, and III

b. I and II

Voltage-gated ion channels serve as site of actions for drugs such as I. Diltiazem and Nifedipine II. Lidocaine and Procaine III. Captopril and Losartan a. I only b. I and II c. III only d. I and III e. I, II, and III

b. I and II

Which of the following is/are important contraindication to the use of antimuscarinis agents or drugs with atropine-like properties? I. History of prostatic hyperplasia II. Glaucoma III. Hypertension a. I only b. I and II c. III only d. I and III e. I, II, and III

b. I and II

Which of the following mechanism explains the blood pressure-lowering effect of thiazide diuretics? I. Reduction in blood volume and cardiac output II. Increased vascular compliance III. Decreased proximal tubular reabsorption of water and sodium a. I only b. I and II c. III only d. I and III e. I, II, and III

b. I and II

Which of the following is/are correct clinical indication for the drug Terbutaline I. Subcutaneously given as management of recalcitrant acute exacerbations of bronchial asthma II. To increase heart rate in symptomatic bradycardia III. Adjunct treatment of malignant hypertension a. I only b. I and II c. III only d. I and III e. I, II, and III

b. I andII

A 45-year old male admitted for severe chest painwas diagnosed with AMI. He was being given morphine 2mg IV for relief of the chest pain. Which of the following is/are important findings that must be observed as expected consequence/s of the use of Morphine? I. Tachycardia II. Hypotension III. Vasospasm a. I only b. II only c. I and III d. II and III e. I, II, and III

b. II only

Which of the following are important considerations regarding the proper handling of Nitroglycerin tablets I. Formulations used in the medicine are explosive II. Sublingual tablets lose their potency when stored in plastic containers III. These are sensitive to light a. I only b. II only c. I and II d. II and III e. I, II, and III

b. II only

Which of the following drugs or drug combination is/are useful in the management of chronic stable angina pectoris? I. Regular release Nifedipine capsule II. Metoprolol tablet III. Metoprolol tablet + Diltiazem tablet a. I only b. II only c. I and III d. II and III e. I, II, and III

b. II only

Which of the following effects may be expected as a direct consequence of blockade of the H1 receptots I. Sedation II. Resolution of initial allergic response III. Anti-nausea and Anti-emetic effect (anti-motion sickness) a. I only b. II only c. I and III d. II and III e. I, II, and III

b. II only

Poisoning resulting from ingestion of high dose quarternary antimuscarinic agents can be best managed with which of th following agents? a. Physostigmine b. Neostigmine c. Policarpine d. Atropine e. Scopolamine

b. Neostigmine

Which of the following interventions may be beneficial in the management of CHF? I. Replacement of chronic digoxi dose with ACE inhibitors II. Add-on therapt with Aldosterone antagonist on patients already on digoxin or ACE-inhibitors III. Use of calcium channel blockers in patient who cannot recieve beta-blockers a. I only b. II only c. I and III d. II and III e. I, II, and III

b. II only

Which of the following statement/s is correctly describe/s the pharmacokinetics of Aspirin? I. Aspirin can be absorbed in the stomach and the small intestines in the hydrolyzed form II. Elimination of aspirin can be best descrined as following a Michaelis-Menten kinetics III. Acidification of the urine facilitates renal elimination of Salicylates a. I only b. II only c. I and III d. II and III e. I, II, and III

b. II only

Which of the following statements correctly describe/s the appropriate use of drugs for CHF I. Beta-blockers when used for patients with stable CHF can be given at doses similar to the dose used for hypertension II. ACE inhibitor dose must be carefully titrated to the maximum allowable or tolerable dose III. Spironolactone has been found to be most useful as initial drug therapy for CHF even prior to the use of ACE inhibitors or casodilators a. I only b. II only c. I and III d. II and III e. I, II, and III

b. II only

Which of the following statements is/are correct regarding the anti-platelet aggregation effect of Aspirin? I. Discontinuation of Aspirin immediatel rrestores platelet aggregation in matter of 1-2 days II. The mechanism involves irreversible actylation of the cyclooxygenase enzyme in platelets III. The effect is clinically significant in all dose ranges for Aspirin a. I only b. II only c. I and III d. II and III e. I, II, and III

b. II only

What is the mechanism of action of Nedocromil? a. Increase outward potassium conductance leading to hyperpolarization of mast cell membrane b. Increase inward chloride conductance leading to hyperpolarization of mast cell membrane c. Increase inward sodium conductance leading to depolarization of mast cell membrane d. Increase outward calcium conductance leading to repolarization of mast cell membrane e. Increase inward potassium conductaance leading to depolarization of mast cell membrane

b. Increase inward chloride conductance leading to hyperpolarization of mast cell membrane

What is the mechanism of action of Inamrinone and Milrinone in the management of heart failure? a. Increase the levels of cAMP by activating the enzyme adenylyl cyclase b. Increase the levels of cAMP by inhibiting the enzyme phosphodiesterase III c. Increase the levels of cGMP by inhibiting the enzyme guanylyl cyclase d. Inhibit the outward calcium flux leading to increased intracellular calcium e. Primarily cause peripheral vasodilation leading to reduced cardiac preload

b. Increase the levels of cAMP by inhibiting the enzyme phosphodiesterase III

What is the primary mechanism of action of the drug Indapamide? a. Inhinition of the 2Cl-Na-K cotransporters in the convoluted tubule b. Inhibition of the Na-Cl contransporters at the convoluted tubule c. Inhibition of the Na-Cl cotransporters at the collecting duct d. Competitive inhibition of aldosterone binding to its receptors e. Competitive inhibition os vasopressin binding to V2 receptors

b. Inhibition of the Na-Cl contransporters at the convoluted tubule

Which of the following mechanism explain the effects of the drug Torsemide and Bumetanide? a. Inhibition of carbonic anhydrase at the proximal convoluted tubule b. Inhibition of the Na-K-2Cl co-transporter at the thick ascending limb of the Loop of Henle c. Inhibition of the Na-Cl co-transporter at the thin ascending limb of the Loop of Henle d. Inhibition of the Na-Cl co-transporter at the distal convoluted tubule e. Inhibition of the Na-Cl co-transporter at the distal tubule and collecting duct

b. Inhibition of the Na-K-2Cl co-transporter at the thick ascending limb of the Loop of Henle

What is the mechanism of action of Sildenafil, a drug popularly known as viagra? a. Inhibits phosphodiesterase (PDE) leading to increase cAMP b. Inhibits PDE leading to increase cGMP c. Stimulates adenylyl cyclase leading to increase cAMP d. Stimulates guanylyl cyclase leading to increase cGMP e. Inhibits phosphodiesterase leading to increase cGMP

b. Inhibits PDE leading to increase cGMP

What is the primary drawback with phenprocoumon that limit its clinical use in the management of thrombosis? a. Ist metabolism to a hepatotoxic metabolite b. Ita long half-life c. Its prominent gastrointestinal side effects d. Its high degree of protein binding e. The long delay in its onset of action

b. Ita long half-life c. Its prominent gastrointestinal side effects

Which of the following NSAIDs is most useful as an analgesic and has been used successfully to replace morphine in some situation involving mild to moderate post surgical pain? When used with an opioid, it may decrease the opioid requirement by as much as 25-50%. a. Naproxen b. Ketorolac c. Tenoxicam d. Diclofenac e. Mefenamic acid

b. Ketorolac

What is the drug of choice for the management of sustained ventricular arrythmia associated with acute myocardial infarction? a. Procainamide b. Lidocaine c. Amiodarone d. Verapamil e. Adenosine

b. Lidocaine

What is the preferred drug for the management of ventricular arrythmias associated with digitalis intoxication? a. Procainamide b. Lidocaine c. Amiodarone d. Verapamil e. Adenosine

b. Lidocaine

Which of the following is an accepted indication for Epinephrine? a. Bronchodilator for intermittent bronchial asthma b. Local vasoconstrictor c. Anti-arrythmic for supraventricular tachycardia d. Tocolytic agent for premature labor e. Inotropic for cardiogenic shock

b. Local vasoconstrictor

Competitive antagonist of the receptors of Dopamine and Benzodiazepine are expected to produced which effect? a. Reversal of the intrinsic activity of the receptors b. Maintain the consitutive activity of the receptors c. Stimulate the constitutive activity of the receptors d. Stimulate the constitutive activity of the recptorsin the absence of any ligand e. Produce a negative efficacy

b. Maintain the consitutive activity of the receptors

The activity of this drug is attributed to its metabolite which is a false neurotransmitter at the central nervous system a. Guanethidine b. Methyldopa c. Propanolol d. Labetalol e. Phenylpropanolamine

b. Methyldopa

Ambenonium, neostigmine, and pyridostigmine are most commonly used for which condition? a. Atropine overdose b. Myasthenia gravis c. Bronchial asthma d. COPD e. ADHD

b. Myasthenia gravis

Phenobarbital when given to a patient on chronic warfarin is classified as a. Pharmacodynamic antagonist b. Pharmacokinetic antagonist c. Physiologic antagonist d. Synergistic agonist e. Additive agonist

b. Pharmacokinetic antagonist

A drug molecule does not possess biologic activity by itself but it converted in the body to an active metabolite is called a/an a. Orphan drug b. Prodrug c. Prototype drug d. Parent drug e. Metabolic drug

b. Prodrug

A patient on warfarin for the last 4 weeks for DVT has a PT INR of 4 but without evidence of bleeding. What is the most appropriate intervention? a. No intervention as within target INR range b. Reduction of the dose by atleast half c. Adimintration of vitamin K d. Adminitration of protamine sulfate e. Administration of tranexamic acid

b. Reduction of the dose by atleast half

What is described by the equation given as follows: Response=f (EN total . Xa / (Xa + Ka) a. Relationship between occupancy of receptor and the dose of the drug b. Relationship between occupancy of receptor and response to the drug c. Relationship between the dose of the drug and the response to the drug d. Relationship between the number of receptors and the dose of the drug e. Relationship between the response to the drug and the transducer function of the tissues

b. Relationship between occupancy of receptor and response to the drug

As an inotropic agent, Norepinephrine (Levophed) has been shown in some small clinical trials to be most useful in which of the following conditions? a. Cardiogenic shock b. Septic shock c. Acute renal failure d. Acute myocarditis e. Dilated cardiomyopathy

b. Septic shock

Reserpine, an alkaloid from Rauwolfia, can lower the blood pressure and depress the CNS. What enzyme or part of the biosynthetic pathway is inhibited by Reserpine to bring about these effects? a. DOPA decarboxylase b. Storage of catecholamines in the presynaptic vesicles c. Release of Norepinephrine from presynaptic terminals d. MAO enzymes e. Uptake-1 mechanism

b. Storage of catecholamines in the presynaptic vesicles

Which of the following agents is associated with formation of antibodies that can cause development of allergic reaction with subsequent exposure to the drug? a. Urokinase b. Streptokinase c. Alteplase d. Tissue plasminogen Activator e. Tenecteplase

b. Streptokinase

What is the primary limitation with the use of Dihydropyridine Calcium Channel Blockers in the management of angina pectoris? a. Tendency to reduce cardiac preload b. Tendency to cause reflex tachycardia c. Tendency to induce reflex vasospam when stopped d. Tendency to cause peripheral edema e. Tendency to reduce cardiac end-diastolic pressure

b. Tendency to cause reflex tachycardia

Which of the following stetements characterizes competitive antagonism? a. There is a shift of the agonist log concentration-effect curve to the right and downwards b. There is a shift of the agonist log concentration-effect curve to the right without a change on the slope or amplitude c. There is an exponential relationship between dose ratio and antagonist concentration d. There is no competition for the binding sites e. There is a shift of the agonist log-concentration effect curve to the left without a change in slope and amplitude

b. There is a shift of the agonist log concentration-effect curve to the right without a change on the slope or amplitude

Constipation, Urinary retention, tachycardia, mydriasis and anhidrosis are expected effects based on the mechanism of action of which of the following drugs? a. Clonidine b. Trimethaphan c. Reserpine d. Sodium nitropusside e. Nicardipine

b. Trimethaphan

Which of the following effects is caused by Prostacylin? a. Bronchoconstriction b. Vasoldilation of the blood vessels c. Inhibition of gastric acid secretion d. Contraction of the uterus e. Inhibition of renin secretion

b. Vasoldilation of the blood vessels

What is the preferred drug for the management of chronic paroxysmal supreventricular tachycardia? a. Quinidine b. Verapamil c. Amiodarone d. Bretylium e. Adenosine

b. Verapamil

Which of the following calcium channel blockers has greater activity on calcium channels in myocardial tissues than calcium channels in the vascular smooth muscles? a. Diltiazem b. Verapamil c. Nifedipine d. Amlodipine e. Felodipine

b. Verapamil

What is the con centration at steady state of Phenobarbital if the drug is given to a 60kg patient as a 65mg tablet once daily with a bioavailability of 90%? The drug has volume distribution of 0.5liters/KBW and t1/2 of 100hrs a. 0.10mg/L b. 1.50mg/L c. 11.70mg/L d. 22.20mg/L e. 48.20mg/L

c. 11.70mg/L

When the methotrexate is used for the management of rheumatoid arthitis, what is the usual maintenance dosing regimen given to patients? a. 150mg OD b. 15mg OD c. 15mg once a week d. 150mg BID e. 15mg BID

c. 15mg once a week

What is the recommended target protime INR for patients who need warfarin and have prosthetic heart valves? a. 1-3 b. 2-3 c. 3-4 d. 4-5 e. 5-6

c. 3-4

At what usual adult dose of aspirin is its anti-inflammatory effect observed? a. 0.3-2.4g/day b. <0.6-4g/day c. 3.2-4g/day d. <0.325g/day e. At all dose range

c. 3.2-4g/day

Cyclic Adenosine Monophophate (cAMP) is generated as a secondary messenger by the action of the enzyme Adenylyl Cyclase. The substrate for the synthesis of cAMP is a. AMD b. ADP c. ATP d. GTP e. Adenosine

c. ATP

Which of the following correctly describes the mechanism of action of fibrinolytic agents? a. Directly catalyze the proteolysis of fobrin and fibrinogen b. Inhibit the action of thrombin by preventing its binding to fibrin and fibrinogen c. Activate plasminogen to plasmin, a serine protease that catalyzes breakdown of fibrin and fibrinogen d. Inhibit activation of fibrinogen by proteolysis of thrombin e. Activate the endogenous anti coagulants protein C and protein S

c. Activate plasminogen to plasmin, a serine protease that catalyzes breakdown of fibrin and fibrinogen

What is the clinical use of the Dopamine-1 agonist Fenoldopan? a. Chronic treatment of Congestive Heart Failure b. Acute treatment of Pheochromocytoma c. Acute treatment of Hypertension d. Chronis treatment of Supraventricular Tachycardia e. Acute treatment of Congestive Heart failure

c. Acute treatment of Hypertension

Measurement of the serum ALT prior to and during therapy of arrythmia is indicated when which of the following drugs for arrythmia is/are used? a. Propafenone b. Moricizine c. Amiodarone d. Flecainide e. Diltiazem

c. Amiodarone

Methylphenidate and Dextroamphetamine are centrally acting sympathomimetics which have found use in which of the following conditions? a. Manic-depression b. Alcohol withdrawal c. Attention deficit hyperactivity disorder d. Generalized anxiety disorder e. Hypotensive epidoses

c. Attention deficit hyperactivity disorder

Paients who are poor Debrisoquin metabolizers have decreased activity of which enzyme system a. CYP 1A2 b. CYP 2C9 c. CYP 2D6 d. CYP 2C19 e. CYP 3A4

c. CYP 2D6

What is the active metabolite of hydroxyzine which is also available as an H1-antihistamine without significant sedating side-effect? a. Fexofenadine b. Astemizole c. Cetirizine d. Loratidine e. Terfenadine

c. Cetirizine

Drugs with mechanism of action that involves Tubulin-binding leading to loss of function of the structural protien include a. Cyclosporine b. Neostigmine c. Colchicine d. Glibenclamide e. Metformin

c. Colchicine

What is the most common side effect of Enalapril? a. Angioedema b. Interstitial nephritis c. Coughs d. Hypotension e. Drug fever

c. Coughs

What is the major mechanism of interaction between Digoxin and Quinidine? a. Displacement of digoxin from tissue binding site b. Decreased hepatic metabolism of digoxin c. Decresed renal clearance of digoxin d. Increase binding of digoxin with ion transporters in the myocardium e. Increase intestinal absorption of digoxin

c. Decresed renal clearance of digoxin

Chloramphenicol when given to a nenonate or premature baby causes cardiovascular collapse manifestin as the so-called "Gray Baby Syndrome" This toxicity in such subsets of the patients is due to what age-related biologic feature? a. Decreased protien binding b. Decreased urinary excretion of the metabolite c. Deficiency of Glucuronosyl acyl transferase d. Increased metabolism by nitro-reduction e. Increased volume of distribution

c. Deficiency of Glucuronosyl acyl transferase

Which of the following agents is pure anteriolar vasodilaator? a. Prazosin b. Sodium Nitropusside c. Diazoxide d. Phentolamine e. Pindolol

c. Diazoxide

Which of the following inotropic agents is most useful for paatient with acute heart failure complicated by acute reduction in creatinine clearance? a. Epinephrine b. Norepinephrine c. Dopamine d. Dobutamine e. Digitalis

c. Dopamine

Which of the following agents exhibits pharmacodynamic interaction with the thiazide diuretics leading to a reduced diuretic effect? a. Furosemide b. Amiloride c. Enalapril d. Hydralzine e. Digoxin

c. Enalapril

Which of the following parameters must be measured when a patient on low molecular weight heparin develops bleeding that may be associated with therapy? a. aPTT b. Prothrombin time c. Factor X assay d. Factor IX assay e. Bleeding time

c. Factor X assay

Diacetylmorphine is the chemical name of which of the following opioids? a. Apomorphine b. Morphine c. Heroin d. Codeine e. Meperidine

c. Heroin

Which of the following agents is/are considered useful as single agent in the acute treatment of vasospastic angina pectoris? I. Nitrovasodilators II. Beta-blockers III. Calcium channel blockers a. I only b. II only c. I and II d. II and III e. I, II, and III

c. I and II

Which of the following is/are accepted indication/s for the drug Spironolactone? I. Adjunct in the management of CHF II. Conn's Syndrome III. Nephrogenic Diabetis Insidipus a. I only b. III only c. I and II d. II and III e. I, II, and III

c. I and II

In the management of bronchial asthma, which of the following medications is/areconsidered as controller medications? I. Low-dose inhaled glucocorticosteroids II. Short-acting beta-2 agonists III. Long-acting inhaled beta-2 agonists a. I only b. II only c. I and III d. II and III e. I, II, and III

c. I and III

Inotropic activity oh the heart in CHF can be increased by which of the following agents? I. Bipyridines II. ACE inhibitors III. Beta-1 agonists a. I only b. II only c. I and III d. II and III e. I, II, and III

c. I and III

The triple response of Lewis following intradermal injection of Histamine includes which of the following? I. Sensory nerve ending stimulation producing flare II. Erythema caused by contraction of vascular smooth muscles III. Endothelian cell contraction leading to exudation of fluids a. I only b. II only c. I and III d. II and III e. I, II, and III

c. I and III

Which of the following beta-blockers have been found to be most useful as anti-arrythmic agent/s? I. Esmolol II. Metoprolol III. Propanolol a. I only b. II only c. I and III d. II and III e. I, II, and III

c. I and III

Which of the following condition when present may increase of formation of the toxic intermediate Nacetyl-p-iminobenzoquinone with regular doses of acteminophen? I. Patient with viral hepatitis II. Patients on warfarin therapy III. Chronic alcoholics a. I only b. II only c. I and III d. II and III e. I, II, and III

c. I and III

Which of the following correctly differentiate/s COX-1 from COX-2 isozymes? I. COX-1 is widely distributed and constitutively expressed II. COX-1 is expressed upon stimulation by sytokines and growth factors III. COX-2 is expressed by inflammatory and immune cells a. I only b. II only c. I and III d. II and III e. I, II, and III

c. I and III

Which of the following effects observe with acute or chronic use of opioids like morphine can be attributed to stimulatio of the mu-receptors? I. Respiratory depression II. Psychosis III. Miosis a. I only b. II only c. I and III d. II and III e. I, II, and III

c. I and III

Which of the following is/are mechanical effects of digoxin on myocardial tissues I. Increase in contractility of myocardial tissues II. Increase in conduction velocity in the AV node III. A positive inotropic effect a. I only b. II only c. I and III d. II and III e. I, II, and III

c. I and III

Which of the following statement correctly describes the effects of glucocorticosteroids in eicosanoid synthesis? I. Dexamethasone inhibits the expression of the COX-2 gene II. Glucocorticosteroids inhibit the synthesis of annexins leading to decreased activity of Phospholipase C III. Glucocorticosteroids stimulates the syntheis of lipocortins which inhibit the activity of Phospholipase A2 a. I only b. II only c. I and III d. II and III e. I, II, and III

c. I and III

Which of the following statements describe/s the clinical use of Theophylline? I. May reduce the nocturnal attack of bronchial asthma when given as sustained-release preparation II. May improve response to high dose inhaledd beta-2 agonists when given intravenously concurrently with beta-2 agonists during acute exacerbation of asthma III. May improve contratility of the diaphragm improving ventilatory response to hypoxia a. I only b. II only c. I and III d. II and III e. I, II, and III

c. I and III

Which of the following statements is/are correct regarding the drug acetaminophen? I. It is a weak inhibitor of COX-1 and COC-2 isozymes in peripheral tissues II. It has significant anti-inflammatory activity III. It is equivalent to Aspirin as analgesic and anti-pyretic a. I only b. II only c. I and III d. II and III e. I, II, and III

c. I and III

Which of the following statements must be considered with regard to the effect of Aspirin aand other NSAIDs on the gastric mucosa? I. The risk of GI intolerance can be reduced if Aspirin or any other NSAIDs is taken with meals II. With Aspirin, the risk of erosive gastritis is obsereved only when Aspirin is given at doses >0.325g/day III. Gastric irritation is most prominent among NSAIDs that inhibit COX-1 more than COX-2 a. I only b. II only c. I and III d. II and III e. I, II, and III

c. I and III

Warfarin exerts its effect primarily by inhibiting the gamma-carboxylation of the glutamate residues of which of the following clotting factors? a. IXa,Xa, XIa, XIIIa b. IIa, VIIa, IXa, Xa c. II, VII, IX, X d. IX, X, XI, XIII e. I, II, V, VII

c. II, VII, IX, X

Which of the following effects can be attributed to atropine? I. Ciliary muscle contraction II. Contraction of the papillary constrictor muscles III. Blockade of vagal slowing of the heart a. I only b. I and II c. III only d. I and III e. I, II, and III

c. III only

Which of the following mechanism may explain the ability of beta-blockers to reduce blood pressure in hypertension? a. Increased compliance of the capacitance vessels b. Relaxation of the primary resistance vessels c. Inhibition of renin release d. Direct inhibition of Angiotensin II receptors e. Decrease in total body stores of sodium

c. Inhibition of renin release

Astemizole and Terfenadine have been withdrawn from the market because of their intrinsic cardiotoxicity. Tha risk of cardiotoxicity is increased with concominant intake of Grapefruit juiceor Erythromycin. The explanation for this interaction is a. Displacemnet from the protien binding of Astemizole or Terfenadine by Grapefruit juice of Erythromycin b. Decreased urinary excretion of Astemizole and Terfenadine c. Inhibtion of the CYP3A4 enzyme by Grapejuice or Erythromycin d. Increased absorption of Astemizole or Terfenadine e. Induction of metabolism of Astemizole and terfenadineby Grapefruit juice or Erythromycin

c. Inhibtion of the CYP3A4 enzyme by Grapejuice or Erythromycin

According to the Fick's Diffusion Equation, the rate of transport of a drug by passive diffusion is a. Independent of the concentration gradient b. Inversely proportional to the membrane surface area c. Inversele proportional to the membrane thickness d. Inversely proportional to the partition coefficient e. Independent of the diffusion constant of drug

c. Inversele proportional to the membrane thickness

Which of the following agents is classified as a selective direct-acting beta-2 agonist? a. Propanolol b. Mathoxamine c. Isoxsuphrine d. Prazosin e. Ephedrine

c. Isoxsuphrine

Which of the following anticoagulant is produced by recombinant DNA technology from the gene that encodes the natural anticoagulant derived from medicinal leeches? a. Dicumarol b. Hirudin c. Lepirudin d. Heparin e. Dalteparin

c. Lepirudin

Chinese patients are most likely to be more sensitiveto effects of which of the following drugs for hypertension that lower doses may have to be used? a. Prazosin b. Amlodipine c. Metoprolol d. HCTZ e. Enalapril

c. Metoprolol

Which of the following drugs if combined with Minoxidil may minimize latter drug's side effect? a. Hydralazine+HCTZ b. Guanethidine+Reserpine c. Metoprolol+Chlorthalidone d. Amlodipine e. Amlodipine+Prazosin

c. Metoprolol+Chlorthalidone

Which of the listed opioid below is along the recommended drugs for the management of acute pulmonary edema? a. Nalbuphine b. Codeine c. Morphine d. Meperidine e. Apomorphine

c. Morphine

All the know NSAIDs are weak acids. Which of the following drugs is the only exception? a. Piroxicam b. Sulindac c. Nabumetone d. Ketorolac e. Tolmetin

c. Nabumetone

What is thw physiologic event that follows stimulation of the NM type cholinergic receptors? a. Opening of inward chloride channels b. Opening of outward potassium channels c. Opening of inward sodium channels d. Opening of the outward calcium channels e. Opening of the outward sodium channels

c. Opening of inward sodium channels

Which of the following eicosanoids has an analogue that is primary indicated as cytoprotectant in peptic ulcer disease? a. PGD2 b. PGI2 c. PGE1 d. PGE2 e. PGF2-alpha

c. PGE1

Sumatriptan is contrindicated in which subset of patients? a. Patients with broncospastic disease b. Patients with acute attacks of migraine c. Patients with ischemic heart disease d. Patients with hypertension e. Patients with Wolff-Parkinson-White syndrome

c. Patients with ischemic heart disease

Omeprazole can inhibiy gastric acid secretion induced by Histamine and Acetylcholine. It is thus classified as what type of antagonist? a. Pharmacodynamic antagonist b. Pharmacokinetic antagonist c. Physiologic antagonist d. Chemivall antagonist e. Non-competitive antagonist

c. Physiologic antagonist

Guanfacine and Guanabenz are primarily agonists at what receeptors? a. Muscarinic-1 b. Nicotinic-N c. Presynaptic alpha-2 d. Postsynaptic alpha-2 e. Alpha-1

c. Presynaptic alpha-2

What is the primary mechnism of action of the drug Amiodarone? a. Prolong the action potential with block of the sodium channels b. Shorten the action potential with block of the sodium channels c. Prolong the action potential with block of the potassium channels d. Shorten the action potential with block of the potassium channels e. Inhibit intrinsic sympathetic stimulation by preventing activation of the enzyme adenylyl cyclase

c. Prolong the action potential with block of the potassium channels

Which of the following drugs is a sodium channel blockers that can prolong the duration of the action potential and dissociates from the channel with intermediate kinetics? a. Lidocaine b. Flecainide c. Quinidine d. Bretylium e. Diltiazem

c. Quinidine

Which of the following side effects is/ are common to the three drugs: Hydralazine, Diazoxide, and Minoxidil? a. Hypertrichosis b. Glucose intolerance c. Reflex tachycardia d. Lupus-like side effect e. Dyslipidemia

c. Reflex tachycardia

Ipratropium, Oxytropium, and Tiotropium and antimuscarinic agents that are most useful as a. Mydriatics b. Cycloplegic agents c. Relaxants of bronchial smooth muscles d. Inhibitors of involuntary muscle contractions e. Inhibitors of bronchial gland secretions

c. Relaxants of bronchial smooth muscles

What is the mechanism of skeletal muscle paralysis produces by Succinylcholine? a. Competitively inhibits binding of acetylcholine to the NM receptors b. Increased inward chloride conductance leading to hyperpolarization of the neuromuscular endplate membrane c. Sustained depolarization of the motor end plate making it unesponsive to other impulses upon repolarization cannot easily depolarized d. Repeated short depolarizations of the motor end plate leading to prolonged state of sodium channel inactivation e. Irreversible and non-competitive inhibition of the NM receptors in both the ganglia and motor end plate with attenuation of depolarizing impulses

c. Sustained depolarization of the motor end plate making it unesponsive to other impulses upon repolarization cannot easily depolarized

Which of the following drugs for bronchial asthma can be given subcutaneously during acute episodes of bronchospasm? a. Salnutamol b. Isoproterenol c. Terbutaline d. Budesonide e. Theophylline

c. Terbutaline

Which of the following mechanisms explains best the action of drug methyldopa? a. The molecule releases norepinephrine in the presynaptic vesicles and is released by nerve stimulation to interact with postsynaptic adrenergic receptors b. The molecule is converted first to alpha-methyldopamine and alpha-methylnorepinephrine which replace norepinephrine in the presynaptic vesicles and are released by nerve stimulation to interact with postsynaptic adrenergic receptors c. The molecule is converted first to alpha-methyldopamine and alpha-norepinephrine which stimulate central alpha receptors d. The molecule is converted first to alpha-methyldopamine and alpha-norepinephrine which inhibit central alpha receptors e. The molecule directly inhibits peripheral alpha and beta receptors

c. The molecule is converted first to alpha-methyldopamine and alpha-norepinephrine which stimulate central alpha receptors

What is the clinical use of the drugs that primarily stimulate the 5-HT1B and 5-HT1D receptors? a. Appetite suppressants b. Anti-emetic c. Treatment of acute migraine attack d. Migraine prophylaxis e. Motility enhancing agent

c. Treatment of acute migraine attack

What is the amino acid precursor in the synthesis of catecholamines? a. Phenylalanine b. Glycine c. Tyrosine d. Tryptamine e. Glutamic acid

c. Tyrosine

What is the primary indication for the drug methadone? a. Analgesic substitute for Morphine b. Antitussive substitute for Codeine c. Weaning off patients addicted to Heroin or Morphine d. Management of biliary colic e. Anesthetic premedication

c. Weaning off patients addicted to Heroin or Morphine

A given drug has a biologic half life of 4 hours. How much of the drug will remain in the body 12 hours after a single dose? a. 75% b. 50% c. 25% d. 12.5% e. 6.25%

d. 12.5%

Which of the following oral doses of morphine is quivalent to 10mg of an IV dose? a. 200mg b. 180mg c. 100mg d. 40mg e. 10mg

d. 40mg

What should be the rate of infusion of Theophylline if the desired plasma concentration is 10ug/mL in a 70 kg patient?Theophylline has a clearance of 0.6mL/minute per kg body weight. a. 6ug/min b. 4.2ug/min c. 17ug/min d. 420ug/min e. 1167ug/min

d. 420ug/min

A given drug with a half life of 8 hours is being administered at 100mg every 8 hours. What is the amount remaining in the body after 32nd hour, just before administering the next dose? a. 50mg b. 75mg c. 87.5mg d. 93.75mg e. 100mg

d. 93.75mg

The activity of regular heparin when given intravenously is best monitored atleast every 6 hours for the first 24 hours of therapy using what parameter? a. Protime b. Platelet count c. Clot reaction time d. Activated partial thromboplastin time e. Bleeding time

d. Activated partial thromboplastin time

Bleeding associated with the use of regular heparin is best managed with which of the following interventions? a. Immediate stopping of therapy b. Administration of Tranexamic acid c. Administration of vitamin K d. Administration of protamine sulfate and stopping therapy e. Lowering of the current dose without stopping therapy

d. Administration of protamine sulfate and stopping therapy

Tacrine, Donepezil, Rivastigmine and Galantamine are indicated for which of the following condition? a. Supraventricular tachycardia b. Curare-induced muscle paralysis c. Accomodative esotropia d. Alzheimer's disease e. Glaucom

d. Alzheimer's disease

What is the primary mechanism of action of the drug Ondansetron and Granisetron? a. Partiaal agonist effect at the 5-HT1A receptor b. Antagonist effect at the 5-HT1B receptor c. Full agonist effect at the 5-HT2A receptor d. Antagonist effect at the 5-HT3 receptor e. Partial agonist effect at the 5-HT4 receptor

d. Antagonist effect at the 5-HT3 receptor

A dominant alpha-1 agonist effect is expected when Epinephrine is given as an IV infusion at what doses? a. <10ng/kg/min b. 10-50ng/kg/min c. 10-80ng/kg/min d. At least 120ng/kg/min

d. At least 120ng/kg/min

What is the dominant adrenergic receptor in the heart? a. Dopamine-1 b. Alpha-1 c. Alpha-2 d. Beta-1 e. Beta-2

d. Beta-1

Digibind which effetively terminates the action of Digitalis glycosides is classified as what type of antagonist? a. Pharmacodynamic antagonist b. Pharmacokinetic antagonist c. Physiologic antagonist d. Chemical antagonist e. Non-competitive antagonist

d. Chemical antagonist

Tolerance is an expected consequence with chronic use of most of the available opioids. Which of the following effects of opioids is tolerance NOT observed? a. Respiratory depression b. Analgesia c. Euphoria d. Constipation e. Sedation

d. Constipation

Which of the following feature common to all carrier-mediated transport processes? a. Movement is along a concentration gradient b. Involves non-specific binding to carriers c. Can be characterized by allosteric inhibition d. Displays a Michaelis-Menten Kinetics e. Non-energy requiring

d. Displays a Michaelis-Menten Kinetics

Which of the following findings is consistent with the skeletal muscle relaxant effect of Pancuronium and Rocuronium? a. Effect is enhanced with administration of succinylcholine b. Effect is reduced with co-administration of tubocurarine c. Initially cause depolarization of the motor end plate leading to fassiculations d. Effect is antagonized by Neostigmine e. Sustained response to tetanic stimulation

d. Effect is antagonized by Neostigmine

Which of the following findings maybe consistent with an overdose of or accidental exposure to an organophosphate a. Constipation b. Mydriasis c. Peeripheral vasodilation d. Emesis e. Decreased sweating

d. Emesis

Which of the following beta-blockers is considered cardio selective? a. Labetalol b. propanolol c. pindolol d. Esmolol e. Timolol

d. Esmolol

Dimenhydrinate and Diphenhydramine are H1 antihistamines that belong to what chemical class of agents? a. Alkylamines b. Phenothiazine c. Piperazine d. Ethanolamine e. Ethylaminediamine

d. Ethanolamine

Which of the following H1 anti-histamine chemical groups possess significant atropine-like effect? a. Alkylamines b. Phenothiazine c. Piperazine d. Ethanolamine e. Piperidines

d. Ethanolamine

Biotransformation of orally administerd drugs occuring primarily in the liver that reduces the amount of drug finally entering the systemic circulation is often referred to as __________. a. Enterohepatic recirculation b. Gastric emptying c. Elimination d. First-pass effect e. Enzyme induction

d. First-pass effect

A male patient wishes to purchase an over-the-counter tablet for colds. Which of the following data you gathered during an initial interview of the patient should prevent you from dispensing the medication? I. The patient is currently taking Nifedipine GITTS II. The patient is currently on Tranylcypromine III. The patient is an elderly with history of urinary hesitancy, intermittency, and frequency a. I only b. I and II c. III only d. I and III e. I, II, and III

d. I and III

In most patients admitted to atropine overdose, most poison control experts recommend which of the following measures as primary management? I. Temperature control with cooling blankets II. Physostigmine as initial therapy III. Diazepam for seizure control a. I only b. I and II c. III only d. I and III e. I, II, and III

d. I and III

Which of the following adrenergic agents is/are most useful for hypodynamic septic shock with hypotension I. Dopamine II. Dobutamine III. Norepinephrine a. I only b. I and II c. III only d. I and III e. I, II, and III

d. I and III

Which of the following effects is are consistent with direct stimulationof the muscarinic receptors? I. Peripheral vasodilation II. Relaxation of the ciliary muscles III. Negative dromotropic effect a. I only b. I and II c. III only d. I and III e. I, II, and III

d. I and III

Which of the following effects is/are consistent with the systemic use of the drug Clonidine? I. Initial transient increase in blood pressure II. "First-dose" phenomenon III. Lowering of the blood pressure a. I only b. I and II c. III only d. I and III e. I. II, and III

d. I and III

A 45 years old female in clonidine 1.0mg/day for thee last four weeks was brought to the hospital for 1 day history of nervousness, tachycardia and abrupt increase in her blood pressure to 180/130mmHg. Her last intake of clonidine was last 2 days prior to admission. Which of the following is/are appropriate interventions that you can suggest for the patient? I. Start patient on Propanolol 50mg BID II. Re-institute Clonidine 1.0mg/day III. Start Labetalol 200mg/day a. I only b. III only c. I and II d. II and III

d. II and III

In salicylate poisoning, serum salicylate levels of 50-80mg/dL can present with which of the following clinical findings? I. Hyperthermia II. Respiratpry alkalosis III. Tinnitus and Vertigo a. I only b. II only c. I and III d. II and III e. I, II, and III

d. II and III

In the long-term management of hypertensiion, which of the following agents is/are considered as appropriate maintenance therapy? I. Regular Nifedipine capsules II. Enalapril tablets III. Indapamide tablets a. I only b. II only c. I and II d. II and III e. I, II, and III

d. II and III

In the management of acute gouty attack, which of the following statement/s is/are correct? I. Uricosuric agent are given at the onset of acute attack to reduce the duration of the episode II. Indomethacin is preffered for the management of the pain of an acute attack III. Colchine relieves the pain and inflammation of an acute attack in 12-24 hours a. I only b. II only c. I and III d. II and III e. I, II, and III

d. II and III

In the management of hypertensive emergencies, which of the following goals or interventions is/are correct? I. Rapid normalization of the blood pressure within 24 hours II. Lowering of the mean arterial pressure by about 25% III. Initiation of oral hypertensives early as soon adequate blood pressure lowering has been achieved a. I only b. II only c. I and II d. II and III e. I, II, and III

d. II and III

In which of the following conditions are ACE-inhibitors found to be clinically useful as first-line treatment? I. Angina pectoris II. CHF III. Diabetic Nephropathy a. I only b. II only c. I and II d. II and III e. I, II, and III

d. II and III

What interaction is present with concominant use of warfarin and the pyrazolone NSAIDs? I. Inhibition of urinary excretion of warfarin II. Inhibition of the oxidative metabolism of S-Warfarin III. Displacement of Warfarin from its protein binding a. I only b. II only c. I and III d. II and III e. I, II, and III

d. II and III

What is the primary use of drugs like Inamirone and Milrinone? I. Alternative to digoxin in patients with chronic CHF II. Management for acute heart failure III. Management of exacerbation of chronic CHF a. I only b. II only c. I and III d. II and III e. I, II, and III

d. II and III

Whgich of the following effects can be attributed to Angiotensin II? I. Stimulation of release of nitric oxide II. Stimulation of synthesis and release of Aldosterone III. Stimulation of release of Norepinephrine a. I only b. II only c. I and II d. II and III e. I, II, and III

d. II and III

Which of the following NSAIDs with minimal anti-inflammatory activity is/are primarily indicated as analgesic especially in the management of post-operative pain? I. Diclofenac II. Ketorolac III. Etodolac a. I only b. II only c. I and III d. II and III e. I, II, and III

d. II and III

Which of the following advice should you give to a patient with bronchial asthma regarding the use of medications for his condition based on the current recommendations? I. Oral bambuterol must be taken on PRN basis for acute exacerbations of bronchial asthma II. Inhalation of salbutamol for acute relief of attack mest be used for not more than three times a day III. He must adequately gargle about 5x after each use Fluticasone inhaler a. I only b. II only c. I and III d. II and III e. I, II, and III

d. II and III

Which of the following agents has/have experimental effect as bronchodilator/s? I. Omalizumab II. Nifedipine III. Nitric oxide a. I only b. II only c. I and III d. II and III e. I, II, and III

d. II and III

Which of the following condition may augment the effect of digitalis glycosides in myoccardial tissues? I. Concurrent use of captopril II. Reduced oxygenation of myocardial tissues III. Concurrent administration of calcium gluconate a. I only b. II only c. I and III d. II and III e. I, II, and III

d. II and III

Which of the following drugs may be used safely for the control of hypertension in pregnant patients with pre-eclampsia? I. Magnesium Sulfate II. Methyldopa III. Hydralazine a. I only b. II only c. I and II d. II and III e. I, II, and III

d. II and III

Which of the following is/are the primary indication/s of Digitalis Glycosides? I. First line drug for the initial management of CHF II. Useful in controlling the rate of ventricular response in atrial fibrillation III. Useful in pregnant patients with CHF a. I only b. II only c. I and III d. II and III e. I, II, and III

d. II and III

Which of the following statements is/are correct regarding Arachidonic acid? I. It is 20-carbon known as eicosapentaenoic acid II. It is mobilized from phospholipids by the enzyme phospholipase A2 III. It is released from phospholipids by the action of the enzymes phopholipase C and Diglyceride lipase a. I only b. II only c. I and III d. II and III e. I, II, and III

d. II and III

Which of the following statements is/are correct regarding the use of thrombolytic agents in myocardial infarction? I. Best response is seen if the drug is given within 24 hours from onset of symptoms II. Generally given to patients with ST segment elevation III. Absolute contraindication is a history of stroke of less than 1 year a. I only b. II only c. I and III d. II and III e. I, II and III

d. II and III

What is the principal mechanism of action of cardiac Glycosides? a. Stimulate the release of calcium for the sarcoplasmic reticulum leading to release in an increase intracellular calcium b. Inhibition of the cell membrane outward calcium transporter preventing calcium extrusion c. Stimulation of inward calcium transporter leading to an increase in intacellular calcium d. Inhibition of the Na-K-ATPase leading to increase in intracellular sodium and preventing calcium extrusion e. Inhibition oh the Na-Ca exchanger in the sarcoplasmic reticulum leading to increase in intracellular calcium

d. Inhibition of the Na-K-ATPase leading to increase in intracellular sodium and preventing calcium extrusion

What is the principal mechanism which can explain anti-pyretic effect of Aspirin? a. Peripheral reduction in the levels of prostaglandins b. Peripheral vasodilation c. Inhibition of prostaglandin syntheis at subcortical sites d. Inhibtion of CNS response to interleukin-1 e. Activation of the thermoregulatory sweat glands

d. Inhibtion of CNS response to interleukin-1

Which of the following drugs bind to receptors located at the cell membrane? a. Thyroid hormone b. Prednisone c. Estrogen d. Insulin e. Vitamin D

d. Insulin

What is the primary indication of the anticoagulant derived from medicinal leeches or its recombinant form? a. Anticoagulation when necessary during pregnancy b. When initiatin anticoagulation therapy in patient requiring chronic anticoagulation c. Treatment of and prophylaxis against pulmonary thromboembolism d. Management of thrombosis associated with heparin-induced thrombocytopenia e. In patients with AMI or unstable angina pectoris

d. Management of thrombosis associated with heparin-induced thrombocytopenia

Which of the following opioids is not associated with biliary colic and thus can be used for severe pain associated with this condition? a. Morphine b. Pentazocine c. Fentanyl d. Meperidine e. Codeine

d. Meperidine

In terms of mechanism of action, which of the following drugs most closely resembles that of Clonidine? a. Phenylphrine b. Reserpine c. Guanethidine d. Methyldopa e. Amlodipine

d. Methyldopa

What is the dominant ganglionic receptor found in most sympathetic fibers? a. Alpha-1 b. Alpha-1 c. Beta-1 d. Nicotinic e. Muscarinic

d. Nicotinic

Whiich of the following drug groups have mechanisms of aaction that involve binding to enzymes as target protiens a. Benzodiazepines b. Barbiturates c. Calcium channel blockers d. Non-steroidal anti-inflammatory agents e. Phenothiazine antipsychotics

d. Non-steroidal anti-inflammatory agents

In which subset of patients should Hydralazine be avoided or used with caution? a. Patients with arrythmias b. Patient with hypertension c. Patients with DM d. Patients with Ischemic Heart disease e. Patients with CHF

d. Patients with Ischemic Heart disease

What is the most important role of beta-blockers in the management of angina pectoris a. Causes coronary artery vasodilatation thus useful in the management of acute anginal attacks b. Dilates the epicardial coronary vessels thus useful in patients with ongoing prinzmetal angina c. Reduces myocardial oxygen demand by reducing the total peripheral resistance making it useful for chronic stable angina pectoris d. Reduces myocardial oxygen demand by reducing myocardial perfusion time making it useful for chronic stable angina pectoris e. Reduces myocardial oxygen demand by decreasing myocardial perfusion time making it useful for all types of angina pectoris

d. Reduces myocardial oxygen demand by reducing myocardial perfusion time making it useful for chronic stable angina pectoris

What is the most important mechansim of termination of effects of released Norepinephrine which is the target of action of tricyclic antidepressants and Cocaine a. Degradation of COMT b. Degradation of MAO c. Sequestration into cells d. Reuptake into the presynaptic terminal e. Binding to presynaptic alpha-2 receptors

d. Reuptake into the presynaptic terminal

Which of the following statements is an important consideration regarding the toxicities and side effects of Theophylline? a. Toxicities and side effects do not occur as long as the serum levels of Theophylline do not exceed 20mg/L b. Adults, neonates, and young infants have faster clearance of Theophylline than children c. Clearance may be decreased with inhibition of hepatic enzymes among chronic smokers d. Seizures and arrythmias are more common when serum levels exceed 40mg/L e. Hepatic dysfunction does not alter significantly the clearance of Theophylline and can thus be safely used for patients with liver disease

d. Seizures and arrythmias are more common when serum levels exceed 40mg/L

A 50 years old male patient with CHF was started on the following medications: digoxin 0.25 mg OD, Hydralazine 10mg BID, furosemide 20mg OD, and ISMN 60mg OD. 1 week after initiating therapy, the patient was admitted for blurring of vision and palpitation. He feels nauseated and has problems discriminating colors. Which of the following sourses of action is appropriate? a. Suggest discontinuing Hydralazine and starting patient on amlodipine b. Suggest giving of metoclopramide for the nausea and examining the probable eye problem c. Suggest starting patient on beta-blocker therapy d. Suggest discontinuing digoxin and furosemide and checking for serum K levels for necessary correction e. Suggest discontinuing all medications and starting patient instead on an ACE inhibitor, vasodilator and a thiazide

d. Suggest discontinuing digoxin and furosemide and checking for serum K levels for necessary correction

A number of drugs classified as specificCOX-2 inhibitors like Rofecoxib (Vioxx) have been withdrawn from the market or are marketed with "black box" warning. What is the primary reason for such actions to be taken against these drugs? a. They are associated with severe agranulocytosis b. They cause severe irreversible hepatic necrosis c. They increase the risk of bleeding in patient on Warfarin d. They increase the risk of thrombosis and cardiac deaths e. They cause worsening of DM and Hypertension

d. They increase the risk of thrombosis and cardiac deaths

The preganglionic fibers of the sympathetic nervous system originate from which roots? a. Cranial nerves III, VII, IX, X b. Sacral roots S1- S4 c. Cervical roots C1-C7 d. Thoracolumbar roots

d. Thoracolumbar roots

What type of protien is primarily targeted by Digitalis glycosides when they exert their effect on the heart? a. Receptors b. Ion channels c. Enzymes d. Transporters e. Structural protiens

d. Transporters

Anaphylactoid reaction may be seen as a side effect with which of the following neuromuscular blockers? a. Pancuronium b. Vecuronium c. Gallamine d. Tubocurarine e. Rocuronium

d. Tubocurarine

Which of the following tissues exhibit the greatest sensitivity to nitrovasodilators at the lowest effective doses? a. Arteries b. Arterioles c. Capillaries d. Veind e. Myocardial tissues

d. Veind

Stimulation of Beta-adrenoceptor, a G-protien-linked receptor involves a generation of a secondary messenger known as a. IP3 b. cGTP c. DAG d. cAMP e. Calcium ions

d. cAMP

100ml of aqueous solutionof a given drug has a concentration of 25%.50ml of Octanol was added and after equlibration, the concentration of the drug in the aqueous layer was 5%. What is the apparent lipid-water partition coefficient of the drug?> a. 0.2 b. 2 c. 4 d. 6 e. 8

e. 8

What is considdered as the toxic plasma concentration of digoxin? a. >0.5mg/mL b. >150mgng/mL c. >10ng/mL d. >5ng/mL e. >2ng/mL

e. >2ng/mL

What is the preferred drug for the management of acute episodes of supraventricular tachycardia? a. Quinidine b. Verapamil c. Amiodarone d. Bretylium e. Adenosine

e. Adenosine

Which of the following nitrovasodilators may induce conversion of hemoglobin to methemoglobin (methemoglobinemia) which can lead to cyanosis? a. Nitroglycerin b. Isosorbide mononitrate c. Isosorbide dinitrate d. Tetrathyl trinitrate e. Amyl nitrite

e. Amyl nitrite

A given adrenergic agent induces uterine relaxation and bronchial smooth muscle relaxation. These effects can be attributed to: a. Alpha-1 stimulation b. Alpha-2 stimulation c. Beta-1 inhibition d. Beta-1 stimulation e. Beta-2 stimulation

e. Beta-2 stimulation

Which of the following drugs classified as NSAIDs inhibit/s the COX-2 morre than the COX-1 isozymes? a. Indomethacin b. Ibuprofen c. Mefenamic acid d. Naproxen e. Celecoxib

e. Celecoxib

Orthostatic hypotension and syncope are manifestations of a phenomenon associated with the first dose of which agent? a. Labetalol b. Methyldopa c. Fenoldopam d. Irbesartan e. Doxazosin

e. Doxazosin

Which of the following anti-hypertensive agents may be more appropriate to use for diabetic patients with evidence of renal disease? a. Prazosin b. Amlodipine c. Metoprolol d. HCTZ e. Enalapril

e. Enalapril

Cimetidine, Ketoconazole, and Allopurinol share a common pharmacokinetic property that predisposes patients taking other drugs concominantly with any of these agents to toxicity. What is this property? a. Inhibition of tubular secretion b. Increased systemic absorption c. Protien binding dsiplacement d. Enzyme induction e. Enzyme inhibition

e. Enzyme inhibition

The primary mechanism of neuro transmitter release is accomplished by what process? a. Diffusion b. Carrier-mediated c. Active transport d. Facilitated transport e. Exocytosis

e. Exocytosis

Which side effect of chlorthalidone is prominent especially among the elderly when Chlorthalidone is given at a dose of at leats 50mg/day and significantly reduced when doses are maintained below 50mg/day? a. Hyperuricemia b. Hyperglycemia c. Increased in serum LDL d. Allergic reactions e. Hyponatremia

e. Hyponatremia

Trimethaphan is expected to produce which of the following effects? a. Excessive sweating b. Pupillary constriction c. Diarrhea d. Urinary incontinence e. Hypotension

e. Hypotension

Which of the following ACE-Inhibitors require hydrolysis to the active metabolite/s in the body to produce effects? I. Enalapril II. Ramipril III. Perindopril a. I only b. II only c. I and II d. II and III e. I, II and III

e. I, II and III

Abstinence syndrome is a rebound effect characterized by rhinorrhea, lacrimation, yawning, chills, gooseflesh, hyperventilation, and hostility observed in which of the following situations? I. Abrupt withdrawal of an opioid agonist in a patient taking the drug chronically II. In a newborn of a mother illicitly taking heroin during her pregnancy III. Administration of Nalbuphine on a patient chronically on morphine a. I only b. II only c. I and III d. II and III e. I, II, and III

e. I, II, and III

Epinephrine is considered as the first line for the management of anaphylaxis and anaphylactic shock. Which of the following explain/s the mechanism of action of Epinephrine in anaphylaxis and anaphylactic shock? I. Inhibition of Histamine release by beta-2 receptor activation II. Alpha-1 activation leading to vasoconstriction III. Beta-2 activation leading to bronchodilation a. I only b. I and II c. III only d. I and III e. I, II, and III

e. I, II, and III

In which subset of patients which are ACE-inhibitors avoided as treatment option? I. Patients with bilateral renal artery stenosis II. Patients with hyperkalemia III. Pregnant patients in their 2nd and 3rd trimesters of pregnancy a. I only b. II only c. I and II d. II and III e. I, II, and III

e. I, II, and III

What are known clinical uses beta-2 agonists? I. In the management of premature labor II. As part of the interventions employed for hyperkalemia III. Management of bronchial asthma a. I only b. I and II c. III only d. I and III e. I, II, and III

e. I, II, and III

What side effect/s may be expected with the use of Amiodarone? I. Hypothyroidism or Htperthyroidism II. Fatal pulmonary fibrosis III. Symptomatic bradycardia and heart block particularly in patients with atrioventricular nodal disease a. I only b. II only c. I and III d. II and III e. I, II, and III

e. I, II, and III

Which of the following agents can be used for hypertensive emergencies I. Enalaprilat II. Diazoxide III. Sodium nitropusside a. I only b. II only c. I and II d. II and III e. I, II, and III

e. I, II, and III

Which of the following agents maybe used for the management of glaucoma? I. Piocarpine-Physostigmine combination therapy II. Ecothiophate III. Demecarium a. I only b. I and II c. III only d. I and III e. I, II, and III

e. I, II, and III

Which of the following beta-blockers has/have been proven in clinical trials to be useful in the management os stable congestive heart failure I. Carvedilol II. Bisoprolol III. Metoprolol a. I only b. I and II c. III only d. I and III e. I, II, and III

e. I, II, and III

Which of the following effects can be attributed to alpha-2 receptor stimulation? I. Vasodilation II. Inhibition of catecholamine release III. Vasoconstriction a. I only b. I and II c. III only d. I and III e. I, II, and III

e. I, II, and III

Which of the following groups can relax the brochial smooth muscles and are primarily used as bronchodilators in bronchial asthma? I. Methylxanthines II. Beta-2 agonists III. Anticholinergic agents a. I only b. II only c. I and III d. II and III e. I, II, and III

e. I, II, and III

Which of the following is/are accepted indication/s for the drug Furosemide? I. Infusion-diuresis in hypercalcemia II. Oliguric acute renal failure III. Acute pulmonary edema a. I only b. III only c. I and II d. II and III e. I, II, and III

e. I, II, and III

Which of the following is/are appropriate indication/s of Allopurinol? I. In the management of chronic tophaceous gout II. In preventing massive uricosuria in patients to be given chemotherapeutic agents for acute leukemia III. In patients with recurrent renal urate stones a. I only b. II only c. I and III d. II and III e. I, II, and III

e. I, II, and III

Which of the following is/are expected electrical effect/s of digoxin on myocardial tissues? I. Decrease in the refractory period of atrial muscles II. Increase in the refractory period of the AV node III. A positive inotropic effect a. I only b. II only c. I and III d. II and III e. I, II, and III

e. I, II, and III

Which of the following is/are possible therapeutic classification/s for available opiates and opioids? I. Cough supressants II. Emetics III. Analgesics a. I only b. II only c. I and III d. II and III e. I, II, and III

e. I, II, and III

Which of the following mechanisms may explain the actions of methylxanthines in bronchial asthma? I. May inhibit the late phase allergic reaction occuring about 2-8hours after an acute attack II. May inhibit the enzyme phosphodiesterase leading to an increase in the levels of cAMP III. May antagonize the action of Adenosine a. I only b. II only c. I and III d. II and III e. I, II, and III

e. I, II, and III

Which of the following mechanisms may play a role of anti-inflammatory effect of Aspirin? I. Decreased synthesis of prostaglandins II. Inhibition of neutrophil migration and adherence III. Stabilize iysosomes of neutrophils a. I only b. II only c. I and III d. II and III e. I, II, and III

e. I, II, and III

Which of the following statement correctly describe the mechanism of action of the opioids? I. They mimic the action of endogenous opioid peptides II. The stimulate the release of endogenous opioid peptides III. They bind to receptors like the mu, kappa, and delta a. I only b. II only c. I and III d. II and III e. I, II, and III

e. I, II, and III

Which of the following statement/s is/are correct regarding Mefenamic Acid, a popular NSAID? I. It is more toxic than Aspirin II. The drug should not be used for more than 1 week III. It should not be given to children a. I only b. II only c. I and III d. II and III e. I, II, and III

e. I, II, and III

Which of the following statement/s is/are correct regarding the drug Methotrexate when used for rheumatoid arthritis? I. The primary mechanism of action involves inhibition of AICAR (aminomidazolecarboxamide) transformylase and thymydylate synthase II. Toxicity of the drug may be minimized with the use of Leucovorin III. The drug is contraindicated in pregnancy a. I only b. II only c. I and III d. II and III e. I, II, and III

e. I, II, and III

Which of the following statements correctly describes the characteristics of Indapamide as a drug for hypertension? I. Its diuretic effect is self-limiting, and may not account for its hypertensive effect after about 2-4 weeks of therapy II. Long-term use leads to improved blood vessel compliance III. Has modest vasodilating property a. I only b. II only c. I and III d. II and III e. I, II, and III

e. I, II, and III

Which of the following statements correctly describes the side effects of succinylcholine? I. Negative chonotropic and inotropic effects which can be attenuated by atropine II. Increase risk of emesis among diabetics and obese patients III. Hyperkalemia when given to burn or trauma patients or patients with close head injury a. I only b. I and II c. III only d. I and III e. I, II, and III

e. I, II, and III

Which of the following statements is/are true regarding the drug methyldopa I. Is current clinical use is in the management of hypertension in pregnancy II. It is associated with warm antibody type of immunohemolytic anemia III. Toxic doses are associated with hepatotoxicity a. I only b. I and II c. III only d. I and III e. I, II, and III

e. I, II, and III

Which of the following agents is currently used as the first-line DMARDs in the management of Rheumatoid Arthritis? a. Hydroxychloroquine b. Auranofin c. Penicillamine d. Azathioprine e. Methotrexate

e. Methotrexate

What is the rationale behind the combination of amiloride and Hydrochlorothiazide (HCTZ) a. Minimize hyperglycemia due to HCTZ b. Reduce risk of renal stone formation with HCTZ c. Reduce hyperkalemia with amiloride d. Minimize bicarbonaturia with HCTZ e. Minimize hypokalemia due to HCTZ

e. Minimize hypokalemia due to HCTZ

Which of the following agents is classified as a partial agonist of opioid receptors? a. Meperidine b. Methadone c. Levallorphan d. Fentanyl e. Nalbuphine

e. Nalbuphine

Which of the following agents is/are classified as opioid antagonist/s? I. Nalorphine II. Naloxone III. Naltrexone a. Meperidine b. Methadone c. Levallorphan d. Fentanyl e. Nalbuphine

e. Nalbuphine

Which of the following substances exert a negative feedback control on Tyrosine Hydroxylase which serves as the mechanis for moment-to-moment regulation of the rate catecholamine synthesis? a. Dopamine b. Acetylcholine c. DOPA d. Epinephrine e. Norepinephrine

e. Norepinephrine

In the adrenal medulla, norepinephrine is converted to epinephrine by the action of which enzyme? a. DOPA decarboxylase b. Tyrosine hydroxylase c. Cathecol-0- nethyl transferase d. Monoamine oxidase e. Phenylethylamine N-methyl transferase

e. Phenylethylamine N-methyl transferase

An administration of one drug leads to a shift in the log dose-response curve of another drug to the left a range in the maximal efficacy, this is called: a. Agonism b. Partial agonism c. Non-competitive antagonism d. Competitive antagonism e. Potentiation

e. Potentiation

Which of the following anti-arrythmic agents has been associated with lupus-like side effects? a. Quinidine b. Lidocaine c. Propafenone d. Amiodarone e. Procainamide

e. Procainamide

Which of the following eicosanoids is a product of the metabolism of Arachidonic acid by the cyclooxygenase enzyme? a. Leukotrienes b. Isoprostanes c. Epoxides d. Lipoxins e. Prostaglandins

e. Prostaglandins

A 44-year old patient is started a week ago on the following medications: Metoprolol 100mg BIB, Diltiazem 30mb tablet BID and Indapamide 1.5 mg OD for poorly controlled hypertension and chronic stable angina pectoris. She is now admitted at the ICU for dizziness and two episodes of syncope. Her heart rate was 45 beats/min. What course of action would you suggest? a. Continue her medication and suggest for a 12-lead ECG b. Stop Metoprolol and request for a 12-lead ECG c. Stop Diltiazem and request for a 12-lead ECG d. Stop indapamide and Diltiazem and request for a 12-lead ECG as well as serum potassium measurement e. Stop Metoprolol and Diltiazem and request for a 12-lead ECG

e. Stop Metoprolol and Diltiazem and request for a 12-lead ECG

Which of the following beta-blockers also has peripheral vasodilating effect similar to Prasozin? a. Propanolol b. Betaxolol c. Nadolol d. Bisoprolol e. labetalol

e. labetalol


Related study sets

Microeconomics chapter 16 (Rent, Interest, and Profit)

View Set

INTEGUMENTARY, CIRCULATORY, AND RESPIRATORY SYSTEMS : UNIT 4 TERMS, SELF CHECK/PRACTICE

View Set

Field Training Mission Statements

View Set

Types of individual life insurance (practice quiz questions)

View Set

Ch 6: The Normal Distribution and Other Continuous Distributions

View Set

Anatomy and Physiology Chapter 4

View Set

BIO 105: Ch.6 Reproduction at the Cellular Level

View Set